Tag Archives: Lop9

Đề thi thử vào 10 chuyên toán năm 2023 – Star Education

Thời gian làm bài 150 phút

Đề bài.

Bài 1. (2,5 diểm)
(a) Giải phương trình $3 x^3+x+3+(8 x-3) \sqrt{2 x^2+1}=0$.
(b) Cho phương trinh $(\sqrt{x}+1)\left(x^2-3(m+1) x+2 m^2+5 m+2\right)=0(m$ là tham số). Tìm tất cả các giá trị của m để phương trình có hai nghiệm phân biệt thỏa mãn nghiệm này là bình phương nghiệm kia.
(c) n là số tự nhiên lớn hơn hoạc bằng 4, cho $n$ số thực $a_1 \leq a_2 \leq \cdots \leq a_n$ thỏa mãn $a_1+a_2+\cdots a_n=0$ và $\left|a_1\right|+\left|a_2\right|+\cdots\left|a_n\right|=A$. Chứng minh rằng
$$
a_n-a_1 \geq \frac{2 A}{n}
$$

Bài 2. (1,5 điểm) Xét các số $a, b, c$ khác 0 và đôi một phân biệt sao cho các phương trình sau đây có một nghiệm chung:
$$
a x^3+b x+c=0(1), b x^3+c x+a=0(2), c x^3+a x+b=0(3) .
$$
(a) Chứng minh $a+b+c=0$.
(b) Chứng minh rằng một trong các phương trình này có ba nghiệm (không nhất thiết phân biệt).

Bài 3. $(1,5$ điểm)
(a) Tìm số tự nhiên có hai chũ số sao cho nó bằng tổng bình phương các chũ số của nó.
(b) Tìm tất cả các số nguyên tố p, sao cho p có thể biểu diễn được dưới dạng $\sqrt{\frac{a^2-4}{b^2-1}}$, trong đó a,b là các số nguyên dương.

Bài 4. ( 3,5 điểm) Cho đường tròn $(O ; R)$ và dây cung $B C=R \sqrt{3}$ cố định, $A$ thay đổi trên cung lớn $B C$ sao cho tam giác $A B C$ nhọn. Các đường cao $B D, C E$ cắt nhau tại $H$. Phân giác trong góc $A$ cắt $D E$ và $B C$ lần lượt tại $K, L$.
(a) Tính $\angle B A C$ và $\angle O H C$.
(b) Chứng minh $\frac{A K}{A L}$ không đổi. Tìm vị trí của A để KL lớn nhât, tính giá trị đó theo $R$.
(c) Chứng minh đường thẳng d qua L vuông góc $O A$ tiếp xúc với một đường tròn cố định.
(d) Đường thẳng qua K vuông góc DE và đường thẳng qua L vuông góc $B C$ cắt nhau tại P. Chứng minh AP luôn đi qua một điểm cố định.

Bài 5. (1 điểm) Có 10 viên bi vàng và 10 viên bi xanh được xếp thành một hàng. Chúng minh rằng tồn tại 10 viên bi liên tiếp sao cho số viên bi vàng và xanh bằng nhau.

LỜI GIẢI

ĐỀ VÀ ĐÁP ÁN CHUYÊN ĐỀ TOÁN 9 – STAR EDUCATION

ĐỀ BÀI.


Bài 1.
a) Giải phương trình: $\sqrt{2 x^2+5 x-6}+\sqrt{2 x^2-x+3}=2 x+1$.
b) Giải hệ phương trình: $\left\{\begin{array}{l}3 x=x y z+y+1 \\\ 3 y=y z x+z+1 \\\ 3 z=z x y+x+1\end{array}\right.$.

Bài 2. Cho các số thực $x, y, z$ thỏa $x^2+y^2+z^2=1$. Tìm giá trị nhỏ nhất và lớn nhất của
$$
A=x^3+y^3+z^3-x^4-y^4-z^4 .
$$

Bài 3. Xét phương trình nghiệm nguyên $x^2+y^2+z^2=x y+k z$ theo ẩn $x, y, z$ và tham số nguyên $k$.
a) Giải phương trình khi $k=3$.
b) Chứng minh rằng khi $k=3^n$ với $n \geq 1$, phương trình có đúng 2 nghiệm.

Bài 4. Cho tứ giác $A B C D$ nội tiếp đường tròn tâm $O$ và ngoại tiếp đường tròn tâm $I$. Phân giác ngoài của góc $\angle B A D$ và $\angle A B C$ cắt nhau tại $E$. Phân giác ngoài của góc $\angle A B C$ và $\angle B C D$ cắt nhau tại $F$. Phân giác ngoài của góc $\angle B C D$ và $\angle C D A$ cắt nhau tại $G$. Phân giác ngoài của góc $\angle C D A$ và $\angle D A B$ cắt nhau tại $H$.
a) Chứng minh tứ giác $E F G H$ nội tiếp.
b) Chứng minh $E, I, G$ thẳng hàng và $H, I, F$ cũng thẳng hàng.
c) Gọi $M, N, P, Q$ là các tiếp điểm của đường tròn nội tiếp $(I)$ tại $A B, B C, C D, D A$. Chứng minh rằng $E G$ là trung trực của $N Q$, và $F H$ là trung trực của $M P$.

Bài 5. Cho 9 điểm (khác nhau) nằm trong một hình vuông có cạnh là 1 .
a) Chứng minh rằng ta luôn có thể tìm một tam giác với các đỉnh từ 9 điểm trên sao cho nó có diện tích không quá $\frac{1}{8}$.
b) Chứng minh rằng ta luôn có thể tìm một tam giác với các đỉnh từ 9 điểm trên sao cho nó có diện tích nhỏ hơn $\frac{1}{8}$.

LỜI GIẢI

Bài 1.

a) Giải phương trình: $\sqrt{2 x^2+5 x-6}+\sqrt{2 x^2-x+3}=2 x+1$.
b) Giải hệ phương trình: $\left\{\begin{array}{l}3 x=x y z+y+1 \\\ 3 y=y z x+z+1 \\\ 3 z=z x y+x+1\end{array}\right.$.

Lời giải

a) Điều kiện: $x \geq-\dfrac{1}{2}$ và $2 x^2+5 x-6 \geq 0$, suy ra $x>0$. Phương trình đã cho tương đương
$$ \sqrt{2 x^2+5 x-6}-x+\sqrt{2 x^2-x+3}-x-1=0$$
$$\Leftrightarrow \dfrac{2 x^2+5 x-6-x^2}{\sqrt{2 x^2+5 x-6}+x}+\dfrac{2 x^2-x+3-x^2-2 x-1}{\sqrt{2 x^2-x+3}+x+1}=0$$
$$\Leftrightarrow \dfrac{x^2+5 x-6}{\sqrt{2 x^2+5 x-6}+x}+\dfrac{x^2-3 x+2}{\sqrt{2 x^2-x+3}+x+1}=0$$
Ta thấy $x=1$ là nghiệm. Xét $x \neq 1$, phương trình trên tương đương
$$\dfrac{x+6}{\sqrt{2 x^2+5 x-6}+x}+\dfrac{x-2}{\sqrt{2 x^2-x+3}+x+1}=0$$
Ta chứng minh $\dfrac{6}{\sqrt{2 x^2+5 x-6}+x}>\dfrac{2}{\sqrt{2 x^2-x+3}+x+1}$

hay $ 3 \sqrt{2 x^2-x+3}>\sqrt{2 x^2+5 x-6} \Leftrightarrow 16 x^2-14 x+21>0$
Bất đẳng thức cuối đúng.
Vậy tóm lại, phương trình đã cho có nghiệm duy nhất $x=1$.

Bài 2.

Cho các số thực $x, y, z$ thỏa $x^2+y^2+z^2=1$. Tìm giá trị nhỏ nhất và lớn nhất của
$$
A=x^3+y^3+z^3-x^4-y^4-z^4 .
$$

Lời giải

Từ giả thiết ta có $-1 \leq x, y, z \leq 1$.
Từ đó suy ra $x^3+y^3+z^3+x^2+y^2+z^2=x^2(x+1)+y^2(y+1)+z^2(z+1) \geq 0.$
Dẫn đến $x^3+y^3+z^3 \geq-\left(x^2+y^2+z^2\right)=-1$.
Lại có: $x^4+x^4+y^4-\left(x^2+y^2+z^2\right)=x^2\left(x^2-1\right)+y^2\left(y^2-1\right)+z^2\left(z^2-1\right) \leq 0.$
nên $x^4+x^4+y^4 \leq x^2+y^2+z^2=1$.
Do đó suy ra $A=x^3+y^3+z^3-\left(x^4+x^4+y^4\right) \geq-1-1=-2.$
Đẳng thức xảy ra khi $x=0, y=0, z=-1$ hoặc các hoán vị.
Áp dụng bất đẳng thức $a b \leq \dfrac{a^2+b^2}{2}$ với mọi số thực $a, b$, ta có:
$ x^3=\sqrt{3} \cdot \dfrac{1}{\sqrt{3}} x \cdot x^2 \leq \sqrt{3} \cdot \dfrac{x^2+x^4}{2}=\dfrac{x^2 \sqrt{3}}{6}+\dfrac{x^4 \sqrt{3}}{2}.$
Tương tự, $y^3 \leq \dfrac{y^2 \sqrt{3}}{6}+\dfrac{y^4 \sqrt{3}}{2}, z^3 \leq \dfrac{z^2 \sqrt{3}}{6}+\dfrac{z^4 \sqrt{3}}{2}$.
Từ đây suy ra $A =x^3+y^3+z^3-x^4-y^4-z^4\leq \dfrac{\sqrt{3}}{6}\left(x^2+y^2+z^2\right)+\dfrac{\sqrt{3}-2}{2} \left(x^4+y^4+z^4\right)$
$\leq \dfrac{\sqrt{3}}{6}+\dfrac{\sqrt{3}-2}{2} \cdot \dfrac{\left(x^2+y^2+z^2\right)^2}{3}$
$=\dfrac{\sqrt{3}}{6}+\dfrac{\sqrt{3}-2}{6}=\dfrac{\sqrt{3}-1}{3}.$
Đẳng thức xảy ra khi $x=y=z=\dfrac{1}{\sqrt{3}}$.

Bài 3. Xét phương trình nghiệm nguyên $x^2+y^2+z^2=x y+k z$ theo ẩn $x, y, z$ và tham số nguyên $k$.
a) Giải phương trình khi $k=3$.
b) Chứng minh rằng khi $k=3^n$ với $n \geq 1$, phương trình có đúng 2 nghiệm.

Lời giải

a) Khi $k=3$, ta có phương trình $x^2+y^2+z^2=x y+3 z \Leftrightarrow 3 z-z^2=x^2-x y+y^2 \geq 0 .$
Suy ra $0 \leq z \leq 3$.
Nếu $z=0$ hoặc $z=3$ thì $x=y=0$.
Nếu $z=1$ hoặc $z=2$ thì $x^2-x y+y^2=2$ hay $(x+y)^2=3 x y+2$. Điều này là vô lý vì số chính phương không thể chia cho 3 dư 2 .
Vậy tất cả nghiệm cần tìm là $(0,0,0),(0,0,3)$.

b) Ta chứng minh bằng cách quy nạp theo $n$. Khẳng định đúng với $n=1$. Giả sử khẳng định đúng đến $n \geq 1$, ta chứng minh khẳng định cũng đúng với $n+1$.
Khi $k=3^{n+1}$, phương trình đã cho tương đương: $(x+y)^2+z^2=3 x y+3^{n+1} z: 3$.
Đặt $a=x+y$.
Giả sử $a$ không chia hết cho 3 thì $z$ cũng không chia hết cho 3 , suy ra $
a^2-1, z^2-1 \vdots 3 \Rightarrow a^2+z^2-2 \vdots 3.$ Điều này là vô lý vì $a^2+z^2: 3.$ Vậy $x+y$ và $z$ chia hết cho .
Khi đó $(x+y)^2+z^2: 9$, dẫn đến $x y: 3$.
Kết hợp với $x+y: 3$ ta kết luận được $x, y$ đều là bội của 3 .
Đặt $x=3 x_0, y=3 y_0, z=3 z_0\left(x_0, y_0, z_0 \in \mathbb{Z}\right)$
Có: $x^2+y^2+z^2=x y+3^{n+1} z \Leftrightarrow x_0^2+y_0^2+z_0^2=x_0 y_0+3^n z_0 .$
Theo giả thiết quy nạp, phương trình trên có đúng hai nghiệm. Theo nguyên lý quy nạp, ta được phát biểu đúng với mọi $n \geq 1$.

Bài 4. Cho tứ giác $A B C D$ nội tiếp đường tròn tâm $O$ và ngoại tiếp đường tròn tâm $I$. Phân giác ngoài của góc $\angle B A D$ và $\angle A B C$ cắt nhau tại $E$. Phân giác ngoài của góc $\angle A B C$ và $\angle B C D$ cắt nhau tại $F$. Phân giác ngoài của góc $\angle B C D$ và $\angle C D A$ cắt nhau tại $G$. Phân giác ngoài của góc $\angle C D A$ và $\angle D A B$ cắt nhau tại $H$.
a) Chứng minh tứ giác $E F G H$ nội tiếp.
b) Chứng minh $E, I, G$ thẳng hàng và $H, I, F$ cũng thẳng hàng.
c) Gọi $M, N, P, Q$ là các tiếp điểm của đường tròn nội tiếp $(I)$ tại $A B, B C, C D, D A$. Chứng minh rằng $E G$ là trung trực của $N Q$, và $F H$ là trung trực của $M P$.

Lời giải

a) Biến đổi góc: $$\angle A E B=180^{\circ}-\angle E A B-\angle E B A=\angle B A I+\angle A B I=\dfrac{1}{2}(\angle B A D+\angle A B C) .$$
Tương tự, $\angle D G C=\dfrac{1}{2}(\angle A D C+\angle B C D)$.
Suy ra $$\angle A E B+\angle D G C=\dfrac{1}{2}(\angle B A D+\angle A B C+\angle A D C+\angle B C D)=\dfrac{1}{2} \cdot 360^{\circ}=180^{\circ} .$$
Vậy tứ giác $E F G H$ nội tiếp.

b) Ta có các tứ giác $A E B I, G D I C$ là các tứ giác nội tiếp nên suy ra
$$\angle A I E+\angle A I D+\angle G I D =\angle A B E+\left(180^{\circ}-\angle I A D-\angle I D A\right)+\angle G C D $$
$$=90^{\circ}-\angle A B I+180^{\circ}-\angle I A D-\angle I D A+90^{\circ}-\angle D C I$$
$$=360^{\circ}-\dfrac{1}{2}(\angle B A D+\angle A B C+\angle A D C+\angle B C D)=180^{\circ} .$$
Vậy $E, I, G$ thẳng hàng. Tương tự, ta cũng có $H, I, F$ thẳng hàng.

c) Gọi $X, Y$ lần lượt là giao điểm của $I E, I B$ và $Q N$.
Biến đổi góc:$$\angle B Y N =180^{\circ}-\angle Y B N-\angle B N Q=180^{\circ}-\dfrac{1}{2} \angle A B C-\dfrac{360^{\circ}-\angle Q A B-\angle N B A}{2}$$
$$=-\dfrac{1}{2} \angle A B C+\dfrac{\angle D A B+\angle A B C}{2}$$
$$=\dfrac{1}{2} \angle D A B=\angle B A I=\angle B E I .$$

Suy ra tứ giác $E B Y X$ nội tiếp, dẫn đến $\angle I X Y=90^{\circ}$.
Mà $I Q=I N$ nên ta được $E I$ là đường trung trực của $Q N$, hay $E Q$ là đường trung trực của $Q N$.
Tương tự, $F N$ của là đường trung trực của $M P$.

Bài 5. Cho 9 điểm (khác nhau) nằm trong một hình vuông có cạnh là 1 .
a) Chứng minh rằng ta luôn có thể tìm một tam giác với các đỉnh từ 9 điểm trên sao cho nó có diện tích không quá $\dfrac{1}{8}$.
b) Chứng minh rằng ta luôn có thể tìm một tam giác với các đỉnh từ 9 điểm trên sao cho nó có diện tích nhỏ hơn $\dfrac{1}{8}$.

Lời giải

Trước tiên ta chứng minh bài toán phụ: một tam giác có ba đỉnh nằm trên cạnh hoặc miền trong của một hình chữ nhật thì có diện tích không quá một nửa diện tích hình chữ nhật ấy.
Thật vậy, giả sử tam giác $M N P$ với $M, N, P$ thuộc cạnh hoặc miền trong hình chữ nhật $A B C$.
Xét trường hợp $M, N$ thuộc cạnh hình chữ nhật, không mất tính tổng quát, $M, N$ nằm trên cạnh $A B$.
Khi đó hạ đường cao $P H$ của tam giác $M N P$ thì $$S_{M N P}=\dfrac{1}{2} P H \cdot M N \leq \dfrac{1}{2} B C \cdot M N \leq \dfrac{1}{2} B C \cdot A B=\dfrac{1}{2} S_{A B C D} .$$
Xét trường hợp $M \in A B$. Kẻ đường thẳng qua $M$ song song với $B C$ cắt $C D$ tại $Q$ và cắt đường thẳng $N P$ tại $T$. Nếu $T$ nằm ngoài đoạn $N P$ thì $$
S_{M N P} \leq S_{M T P} \leq \dfrac{1}{2} S_{M Q C B} \leq \dfrac{1}{2} S_{A B C D} .$$
Nếu $T$ thuộc đoạn $N P$ thì $$S_{M N P}=S_{M T N}+S_{M T P} \leq \dfrac{1}{2} S_{M Q D A}+\dfrac{1}{2} S_{M Q C B}=\dfrac{1}{2} S_{A B C D} .$$


Cuối cùng, nếu $M, N, P$ dều không thuộc cạnh hình chữ nhật, không mất tính tổng quát, giả sử $M$ có khoảng cách gần với $A B$ nhất trong ba điểm $M, N, P$, kẻ đường thẳng qua $M$ song song với $A B$ cắt $A D, B C$ tại $R, S$.
Khi đó, $$S_{M N P} \leq \dfrac{1}{2} S_{R S C D} \leq \dfrac{1}{2} S_{A B C D} .$$
Vậy tóm lại, ta luôn có $S_{M N P} \leq \dfrac{1}{2} S_{A B C D}$. Đẳng thức xảy ra khi tam giác có một cạnh, giả sử $N P$ là cạnh của hình chữ nhật và $M$ nằm trên cạnh của hình chữ nhật đối diện với cạnh $N P$.
Trở lại bài toán, chia hình vuông thành bốn hình vuông nhỏ có diện tích là $\dfrac{1}{4}$ bởi hai đường trung bình.
Theo nguyên lý Dirichlet, tồn tại 3 diểm cùng thuộc một hình vuông nhỏ.
Diện tích tam giác tạo bởi 3 điểm này không quá $\dfrac{1}{2}$ diện tích hình vuông nhỏ, tức là không quá $\dfrac{1}{8}$ (nếu 3 điểm thẳng hàng thì ta coi như đó là tam giác có diện tích bằng 0 ).
Mà các điểm nằm bên trong hình vuông dẫn đến không có cạnh nào của tam giác này là cạnh của hình vuông, cho nên diện tích tam giác này phải bé hơn $\dfrac{1}{8}$.
Hoàn tất chứng minh.

Một số bài đường tròn và tiếp tuyến

Bài 1. Cho đường tròn tâm $O$ đường kính $A B$. $C$ là một điểm thuộc đường tròn. $d_1$ và $d_2$ lần lượt là tiếp tuyến tại $A$ và $B$ của $(O)$. Tiếp tuyến tại $C$ cắt $d_1, d_2$ lần lượt tại $D$ và $E$. $B C$ cắt $d_1$ tại $F$.
a) Chứng minh $d_1 | d_2$ và $D$ là trung điểm của $A F$.
b) Vẽ đường cao $C H$. Chứng minh rằng $A E, B D$ và $C H$ dồng quy tại trung điểm của $C H$.
c) Chứng minh $O F \perp A E$.

Lời giải.

a) $d_1$ là tiếp tuyến tại $A$ nên $O A \perp d_1, d_2$ là tiếp tuyến tại $B$ nên $d_2 \perp O B$, mà $O, A, B$ thẳng hàng, suy ra $d_1 / / d_2$.
Ta có $\angle A C B=90^{\circ}$, suy ra $\angle D C F+$ $\angle D C A=\angle D F C+\angle D A C=90^{\circ}$. (1)
Hơn nữa $D A=D C$ (t/c tiếp tuyến), tam giác $D A C$ cân tại $D$, suy ra $\angle D C A=$ $\angle D A C$. (2)
Từ (1) và (2) ta có $\angle D C F=\angle D F C$, tam giác $D C F$ cân tại $D$.
Vậy $D F=D C=D A$, hay $D$ là trung điểm của $A F$.
b) Gọi $I$ là giao điểm của $B D$ và $A E$. Ta có $A D / / B E$ nên $\frac{B I}{I D}=\frac{E B}{A D}(3)$.
Mặt khác do $A D=D C$ và $E B=E C$, suy ra $\frac{E B}{A D}=\frac{E C}{D C}$ (4).

Từ (3) và (4) ta có $\frac{B I}{I D}=\frac{E C}{D C}$, suy ra $I C / / A D$ (Thalet đảo).

Mà $A D \perp A B$ nên $C I \perp A B$, vậy $C, I, H$ thẳng hàng.

Do đó $A E, B E, C H$ đồng quy tại $I$.
Ta có $\frac{C I}{A D}=\frac{E I}{E A}, \frac{I H}{A D}=\frac{B I}{B D}$ và $\frac{E I}{E A}=$ $\frac{B I}{B D}$, nên $\frac{C I}{A D}=\frac{I H}{A D}$, suy ra $I C=I H$ hay
$I$ là trung điểm của $C H$.
c) Ta có $E B \cdot A D=E C \cdot C D=O C^2=R^2$, mà $A F=2 A D$ nên $E B \cdot A F=2 R^2$.

Suy ra $E B \cdot A F=A O \cdot A B$, suy ra $\frac{E B}{A B}=\frac{O A}{A F}$, do đó $\tan E A B=\tan A F O$, suy ra $\angle E A B=$ $\angle A F O$.
Mà $\angle E A B+\angle E A F=90^{\circ}$ nên $\angle E A B+$ $\angle A F O=90^{\circ}$. Do đó $O F \perp A E$.

Bài 2. Cho đường tròn tâm $O$ bán kính $R$. $A$ là một điểm nằm ngoài đường tròn, từ $A$ dựng các tiếp tuyến $A B, A C$ dến $(O)$ với $B, C$ là các tiếp điểm. Một cát tuyết qua $A$ cắt $(O)$ tại $D$ và $E$ trong đó $D$ nằm giữa $A$ và $E$.Gọi $H$ là giao điểm của $O A$ và $B C$.
a) Chứng minh $O H \cdot O A=R^2$.
b) Gọi $M$ là trung điểm của $D E$. Chứng minh 4 điểm $O, M, B, C$ cùng thuộc đường tròn.
c) Tiếp tuyến tại $D$ và $E$ của $(O)$ cắt nhau tại điểm $P$. Chứng minh $P, B, C$ thẳng hàng.

Lời giải.

a) Ta có $A B, A C$ là tiếp tuyến nên $A B=A C$, và $O B=O C=R$, suy ra $O A$ là trung trực của $B C$, suy ra $O A \perp B C$ tại $H$.
Tam giác $O A B$ có $\angle O B A=90^{\circ}$ (t/c tiệp tuyến) và $B H \perp O A$ nên $O H \cdot O A=O B^2=$ $R^2$.
b) $M$ là trung điểm $D E$, suy ra $O M \perp D E$.
Ta có $\angle O B A=\angle O M A=\angle O C A=90^{\circ}$, suy ra 5 diểm $O, M, B, A, C$ cùng thuộc đường tròn đường kính $O A$.
c) Ta chứng minh được $O P \perp D E$, suy ra $O, M, P$ thẳng hàng và $O M . O P=O D^2=$ $R^2$.
Suy ra $O M \cdot O P=O H \cdot O A$, suy ra $\frac{O M}{O H}=$ $\frac{O P}{O A}$.
Xét tam giác $O M A$ và tam giác $O H P$ có:
$\angle A O P$ chung $\frac{O M}{O H}=\frac{O P}{O A}$ $\angle O H P=\angle O M A=90^{\circ}$.
Ta có $B C, P H$ vuông góc với $O A$ tại $H$ nên $P, B, C$ thẳng hàng.

Bài 3. Cho tam giác $A B C$ vuông tại $A(A B<A C)$. Vẽ đường tròn tâm $O$ đường kính $A C$ cắt cạnh $B C$ tại $D$. Gọi $H$ và $K$ lần lượt là trung điểm của hai cạnh $A D$ và $C D$. Tia $O H$ cắt cạnh $A B$ tại $E$. Tia $O K$ cắt đường thẳng $E D$ tại $N$ và cắt đường tròn tâm $O$ tại $I$.
(a) Chứng minh $D E$ là tiếp tuyến của $(O)$.
(b) Chứng minh $O H D K$ là hình chữ nhật.
(c) Chứng minh tia $D I$ là tia phân giác của $\angle N D C$.
(d) Gọi $S$ là giao điểm của $O B$ với $A D$. Từ $S$ vẽ đường thẳng vuông góc với $A O$ và cắt tia $O H$ tại $Q$. Chứng minh 3 điểm $A, Q, N$ thẳng hàng.

Lời giải.

Hình 1

a) $OH$ là trung trực của $AD$, suy ra $EA = ED$. Từ đó $\triangle EDO = \triangle EAO (ccc)$, suy ra $\angle EDO = \angle EAO = 90^\circ$. Do đó $ED$ là tiếp tuyến của $(O)$.

b) Do $K$ là trung điểm $CD$ nên $OK \bot CD$, tứ giác $OHDK$ có $\angle D = \angle H = \angle K = 90^\circ$ nên là hình chữ nhật.

c) Ta có tam giác $ODI$ cân tại $O$ nên $\angle ODI = \angle OID$ (1)
Mà $\angle ODI = \angle ODK + \angle KDI, \angle OID = \angle OND + \angle NDI$ (2)
Và $\angle OND = \angle ODK$ (vì cùng phụ $\angle DON$) (3)
Từ (1), (2), (3) ta có $\angle KDI = \angle NDI$

d) Gọi $L$ là giao điểm $AQ$ và $OS$.
Trong tam giác $ASO$ có $AQ, SQ$ là các đường cao, nên $Q$ là trực tâm, suy ra $AQ \bot OS$ tại $L$. (4)
Ta có $OL \cdot OB = OA^2$
và $OK \cdot ON = OD^2 = OA^2$
Suy ra $\angle OK \cdot ON = OL \cdot OB$
Suy ra $\triangle OLN \backsim \triangle OKB$, suy ra $\angle OLN = \angle OKB = 90^\circ$ (5)
Từ (4), (5) ta có $A, L, N$ thẳng hàng, hay $A, Q, N$ thẳng hàng.

Bài 4. Cho đường tròn $(O ; R)$ và một điểm $S$ nằm ngoài đường tròn $(O)$. Vẽ hai tiếp tuyến $S B, S C$ đến $(O)$ với $B, C$ là hai tiếp điểm. Gọi $H$ là giao điểm của $S O$ với $B C$.
(a) Vẽ đường kính $B A$ của $(O)$. Chứng minh $A C || S O$ và $H B \cdot H C=H O \cdot H S$.
(b) Vẽ đường thẳng $d$ vuông góc vớ $A B$ tại $O$, đường thẳng $d$ cắt đường thẳng $A C$ tại $E$. Chứng minh $S E=R$.
(c) Vẽ $C K$ vuông góc với $A B$ tại $K$. Gọi $I$ là trung điểm của cạnh $C K$. Chứng minh 3 điểm $S, I, A$ thẳng hàng.

Lời giải.

a) Do $AB$ là đường kính của $(O)$ nên $\angle ACB = 90^\circ$. (1)

Ta có $SB = SC$ và $SO$ phân giác $\angle BSC$ nên $SO$ là trung trực của $BC$, do đó $OS \bot BC$ tại $H$.

Từ đó ta có $AC ||OS$ vì cùng vuông góc $BC$.

b) $\triangle AOE = \triangle OBS (gcg)$, suy ra $OE = BS$.

Tứ giác $OESB$ có $OE||BS$ (Cùng vuông góc $AB$), và $OE = BS$ nên $OESB$ là hình bình hành, hơn nữa có $\angle OBS= 90^\circ$ nên là hình chữ nhật, do đó $SE = OB = R$.

c) Ta có $OASE$ là hình bình hành, suy ra $AS$ cắt $OE$ tại trung điểm $T$ của mỗi đoạn.
$CK ||OE$
Gọi $I’$ là giao điểm của $AS$ và $CK$
Ta có $\dfrac{I’K}{OT} = \dfrac{AI’}{AT} = \dfrac{CI’}{ET}$
Mà $OT = ET$ nên $KI’ = CI’$, hay $I’ \equiv I$
Vậy $A, I, S$ thẳng hàng

Bài 5. Cho đường tròn $(O ; R)$ và điểm $M$ ở ngoài đường tròn $(O)$. Kẻ tiếp tuyến $M A, M B$ đến $(O)$ với $A, B$ là hai tiếp điểm. Đường thẳng $A B$ cắt $(O)$ tại $K$.
(a) Kẻ đường kính $A N$ của $(O), B H \perp A N$ tại $H$. Chứng $\operatorname{minh} M B \cdot B N=B H \cdot M O$.
(b) Đường thẳng $M O$ cắt đường tròn $(O)$ tại $C$ và $D(C$ nằm giữa $O$ và $M)$. Chứng minh $O K \cdot M K=C K \cdot D K$.
(c) $E$ đối xứng với $C$ qua $K$. Chứng minh $E$ là trực tâm của tam giác $A B D$.
(d) Chứng minh $\sin \angle M^{\circ} A B=\frac{C K}{A K}+\frac{C K}{A M}$

Lời giải.

a) Chứng minh tam giác $OMB$ và $NBH$ đồng dạng.
b) $OK \cdot MK = AK^2 = KC \cdot KD$
c) $ACBE$ là hình thoi, suy ra $BE||AC$, mà $AC \bot AD$ suy ra $BE \bot AD$
$DE \bot AB$
Do đó $E$ là trực tâm tam giác $ABD$.

d) $\angle CAK = \angle CAM$ (chứng minh ở bài trên)
Do đó $\dfrac{CK}{CM} = \dfrac{AK}{AM}$, suy ra $\dfrac{CK}{AK} = \dfrac{CM}{AM}$
Từ đó $VP = \dfrac{CK}{AK} + \dfrac{CK}{AM} = \dfrac{CM}{AM} + \dfrac{CK}{AM} = \dfrac{KM}{AM} = \sin MAB$

Bài 6. Cho hình vuông $A B C D$ cạnh $a, E$ là cung thuộc cung nhỏ $B D$ của đường tròn tâm tâm $A$ bán kính $a$. Tiếp tuyến tại $E$ cắt $C D$ tại $F$ và $B C$ tại $G$.
(a) Chứng minh chu vi tam giác $C F G$ bằng $2 a$.
(b) $A F, A G$ cắt $B D$ tại $I$ và $H$. Chứng minh $H E=$ $H B, I E=I D$

và $H I^2=D I^2+B H^2$
(c) Chứng minh $F H, G I$ và $A E$ đồng quy.

Lời giải.

a) $CD, CB, FG$ là tiếp tuyến của $(A;a)$
Suy ra $FE = FD, GE = GB$
$P_{CFG} = CF + FG + CG = CF + EF +EG+CG = CF+DF +GB+CG = CD+ CB = 2a$

b) $AF$ là trung trực $DE$, và $AG$ là trung trực $BE$
Suy ra $IE = ID, HB = HE$
$\triangle IEF = \triangle IDF \Rightarrow \angle IEF =\angle IDF = 45^\circ$
Tương tự cũng có $\angle HEG = 45^\circ$
Suy ra $\angle IEH = 90^\circ$
Áp dụng pitago cho tam giác $EIH$ ta có $IH^2 = IE^2 + HE^2 = ID^2 + HB^2$

c) Ta có $AF$ là phân giác $\angle DAE$, $AG$ là phân giác của $\angle BAE$
Suy ra $\angle FAG = \dfrac{1}{2} \angle BAD = 45^\circ$.
$\triangle AIH \backsim \triangle DIF (gg)$, suy ra $IA \cdot IF = ID \cdot IH$
Suy ra $\triangle IFH \backsim \triangle IDA \Rightarrow \angle IFH = \angle IDA = 45^\circ$
Suy $\angle AHF = 90^\circ$ hay $FH \bot AG$.
Chứng minh tương tự $GI \bot AF$.
Tam giác $FG$ có $AE, FH, GI$ là các đường cao nên đồng quy.

Bài 7. (Cuối khóa 1 – Star Education 2018) Cho đường tròn $(O ; R)$ và điểm $A$ nằm ngoài đường tròn. Từ $A$ vẽ các tiếp tuyến $A B, A C$ dến $(O)$ ( $B, C$ là các tiếp điểm). $O A$ cắt $B C$ tại $H$.
a) Chứng minh $O H \cdot O A=R^2$ và 4 điểm $O, A, B, C$ cùng thuộc một đường tròn.
b) Đường tròn tâm $I$ đường kính $A B$ cắt $(O)$ tại điểm $D$ khác $B$. Chứng minh $I D$ là tiếp tuyến của $(O)$.
c) Tiếp tuyến tại $H$ và tại $A$ của $(I)$ cắt nhau tại $P$. Chứng minh $B, D, P$ thẳng hàng.

d) Tiếp tuyến tại $H$ của $(I)$ cắt $O B$ tại $M$; gọi $N$ là trung điểm $P M$, đường thẳng qua $P$ song song $B N$ cắt $A B$ tại $K$. Chứng minh $H K, A M$ và $B D$ đồng quy.

Lời giải.

a)

Xét $\triangle A B O$ vuông tại $B$ có:

$B H$ là đường cao $\Rightarrow O H \cdot O A=O B^2=R^2$ (Hệ thức lượng)

Ta có: $\triangle A B O$ vuông tại $B \Rightarrow A, B, O$ thuộc đường tròn đường kính $A O$. (1)

Lại có $\triangle A C O$ vuông tại $C \Rightarrow A, C, O$ thuộc đường tròn đường kính $A O$. (2)

Từ (1) và (2) suy ra $A, B, O, C$ thuộc đường tròn đường kính $A O$.

b)

Ta có: $\triangle A B D$ nội tiếp đường tròn đường kính $A B \Rightarrow \triangle A B D$ vuông tại $D$

Mà $I$ là trung điểm cạnh huyền $A B \Rightarrow I B=I D$
Ta có: $I B=I D, O B=O D$ nên $I O$ là trung trực của $B D$ $\Rightarrow \angle I B O=\angle I D O=90^{\circ}$ nên $I D$ là tiếp tuyến của $(O)$.

c) Tiếp tuyến tại $H$ và tại $A$ của $(I)$ cắt nhau tại $P$. Chứng minh $B, D, P$ thẳng hàng.

Gọi $E=I P \cap A H$ và $F=I O \cap B D$.
Sử dụng tính chất hai tiếp tuyến cắt nhau và hệ thức lượng, ta chứng minh được
$$
I E \cdot I P=I A^2=I D^2=I F \cdot I O \Rightarrow \frac{I F}{I P}=\frac{I E}{I O}
$$

Từ đó, chứng minh được $\triangle I F P \backsim \triangle I E O$ (c.g.c)
$$
\Rightarrow \angle I E O=\angle I F P=90^{\circ} \text {. }
$$

Ta có: $B D$ đi qua $F$ và vuông góc $I O, F P$ đi qua $F$ và vuông góc $I O$ nên hai đường thẳng này trùng nhau. $\Rightarrow B, D, P$ thẳng hàng.

d)

Chứng minh $I H$ là đường trung bình của $\triangle A B C \Rightarrow I H || A C$. Mà $I H \perp P M$ và $A C \perp O C$.

Suy ra: $H M || O C$. Lại có $H$ là trung điểm $B C$ nên $M$ là trung điểm $O B$.

Gọi $Q$ là giao điểm của $P K$ và $B O$.
Ta có: $B N || P Q$ và $N$ là trung điểm của $P M$ nên suy ra $B$ là trung điểm của $Q M$.

Gọi $J=B P \cap A M$.
Ta có :
$ B Q ||A P \Rightarrow \frac{B K}{K A}=\frac{B Q}{P A}=\frac{B M}{P A} . $
$B M || A P \Rightarrow \frac{B M}{P A}=\frac{B J}{J P}$
Suy ra: $\frac{B K}{K A}=\frac{B J}{J P}$ nên $K J || A P$. Chứng minh tương tự $J H ||A P$. Từ đó ta có $K, J, H$ thẳng hàng.

Vậy $H K, B P, A M$ dồng quy tại $J$.

Bài tập luyện tập.

Bài 6. Cho tam giác $A B C$ nhọn. Các đường cao $A D, B E$ và $C F$ cắt nhau tại $H$. Gọi $M, N$ lần lượt là trung điểm của $B C$ và $A H$.
(a) Chứng minh $N E, N F$ là tiếp tuyến của đường tròn ngoại tiếp tam giác $B C E$.
(b) Chứng minh 5 điểm $D, E, F, M, N$ cùng thuộc một đường tròn.
(c) Gọi $G$ là giao điểm của $A D$ và $E F$. Chứng minh $N G \cdot N D=N A^2$.

Bài 7. Cho nửa đường tròn tâm $O$ đường kính $A B=2 R$. Trên tiếp tuyến tại $A$ của $(O)$ lấy điểm $C$ sao cho $A C=A B$. Từ $C$ vẽ tiếp tuyến $C D$ dến $(O)$ cắt tiếp tuyến tại $B$ ở điểm E.
(a) Tính $B E$.
(b) Đường cao $D F$ của tam giác $A B D$ cắt $B C$ tại $G$. Chứng minh rằng $A, G, E$ thẳng hàng.
(c) Gọi $H$ là giao điểm của $O C$ và $A D$. Tính $\angle D H B$.
(d) Gọi $I$ là giao điểm của $B C$ và $(O)$. Tứ giác $I D B H$ là hình gì? Tại sao?

Bài 8. Cho tam giác $A B C$ nhọn nội tiếp đường tròn $(O) . M$ là trung điểm $B C$. Từ $A$ dựng các tiếp tuyến đến đường tròn $(O ; O M)$ cắt $B C$ tại $D$ và $E$ sao cho $D$ và $C$ khác phía đối với $M ; E, B$ khác phía đối với $M$. Chứng minh rằng các tam giác $A D C$ và $A B E$ cân.

Bài 9. Cho tam giác $A B C$ vuông tại $A, A B=a, B C=2 a$. Đường cao $A H$. Từ $B, C$ vẽ các tiếp tuyến $B D, C E$ dến đường tròn tâm $A$ bán kính $A H$.
(a) Tính $A H$ và số đo $\angle A B C$.
(b) Chứng minh $D, A, E$ thẳng hàng.
(c) Chứng minh $E D$ là tiếp tuyến của đường tròn đường kính $B C$.
(d) Chứng minh $D C, B E$ và $A H$ dồng quy.

Bài 10. Cho hình vuông $A B C D$ cạnh $2 a$, tâm $O$. Đường tròn tâm $O$ bán kính $a$ tiếp xúc với $A B$ và $B C$ tại $E$ và $F$. Gọi $P$ là một điểm trên cung nhỏ $E F$. Tiếp tuyến tại $P$ cắt $A B, B C$ tại $M$ và $N$. Đặt $M B=c, B N=y$.
(a) Chứng minh rằng $x+y+\sqrt{x^2+y^2}=2 a$.
(b) Chứng minh rằng $A M \cdot C N=2 a^2$.
(c) Gọi $K$ là trung điểm của $A D$. Chứng minh rằng $M K |$ $D N$.

Giải toán như … viết văn


Mình đăng lại bài viết của bạn Nguyễn Tiến Hoàng gửi cho Tập san Star Education số 10.

Nếu bạn đang tự hỏi rằng tên bài viết này có nhầm lẫn gì không, thì không hề đâu, bạn đã đọc đúng rồi đấy. Trước khi bắt đầu đọc, hãy lưu ý rằng, bài viết này rất nhiều chữ.
Một trong những vấn đề muôn thuở của học sinh Việt Nam, theo quan sát của người viết bài, là một nỗi sợ vô hình đối với các bài toán tổ hợp trong bất kỳ một kỳ thi lớn hay nhỏ. Tổ hợp ở đây không giới hạn trong phạm vi các bài toán đếm mà mang một nét nghĩa rộng hơn thế, tập trung vào khả năng diễn giải và suy luận. Mỗi bài toán dù trong quá trình luyện tập tại nhà, hay là bước vào thực tế thi cử, đều là một vấn đề hoàn toàn mới lạ với các bạn học sinh. Thông thường có hai hình thức để xoay sở:


a) Giải càng nhiều bài tập càng tốt để thu nhận kinh nghiệm. Đây thực ra không phải điều xấu, nhưng việc lạm dụng quá đà sẽ khiến học sinh chỉ trông đợi vào việc gặp lại những thứ quen thuộc, và thậm chí biến tướng thành việc học thuộc lòng.
b) Tuỳ cơ ứng biến và tin tưởng vào trực giác của bản thân. Điều này cũng thú vị bởi xét cho cùng thì một bài toán trong một kỳ thi ở bậc trung học, dù thi gì đi nữa, cũng chỉ là một vấn đề có thể được giải quyết trong thời gian ngắn, thành ra khả năng lớn là mỗi người sẽ tìm được một cách tiếp cận riêng mang tính sáng tạo. Thế nhưng trong một ngày xấu trời, sự nhạy bén không đồng hành, thì phải làm sao ?

Trong bài viết này, người viết muốn giới thiệu một hướng tiếp cận mang tính chất trung hoà và tập trung vào một khâu mà các bạn học sinh thường bỏ quên: phân tích bài toán. Các phân tích cẩn thận và rõ ràng để dần gỡ rối vấn đề được đặt ra đóng vai trò quan trọng tương tự như dàn ý trong việc viết văn. Điều này trở nên then chốt với các vấn đề phức tạp.
Sự phân tích nên tiến hành ra sao ? Bốn câu hỏi cơ bản sau nên được trả lời:
a) “Có gì ?” Bước đầu tiên không khác việc đọc hiểu là bao. Cần chú ý đến từng câu chữ dù là nhỏ nhất. Việc đọc kỹ các giả thiết được đưa ra giúp người giải toán hình dung được những đối tượng đã xuất hiện trong bài toán.
b) “Cần gì ?” Đây là bước giúp hiểu được yêu cầu của bài toán.
c) “Khó khăn gì ?” Bước này quan trọng nhất và đòi hỏi sự kiên nhẫn. Khi thực hiện cẩn thận hai bước đầu tiên, một số vấn đề sẽ phát sinh rất tự nhiên. Các đối tượng được đưa ra đã rõ ràng hay chưa ? Những giả thiết trong bài toán để làm gì ? Tại sao đề bài lại hỏi như thế ? Liệu các đối

tượng có liên kết gì với nhau ? Cấu trúc của từng thành phần hay cả tổng thể là thế nào ? Và còn nhiều thứ phải chú ý nữa.
d) “Giải quyết thế nào ?” Đây là việc trả lời các câu hỏi trên một cách trực tiếp. Việc đặt ra các câu hỏi tự nhiên trong bước trên sẽ giúp người giải toán nhận ra những gì cần thực hiện. Một nguyên tắc chung là, hãy phân tích và liên tục đặt câu hỏi để giảm sự phức tạp, đến khi mọi thứ có thể diễn giải được thật dễ hiểu. Việc gõ rối cần đi từ nội tại từng đối tượng (chẳng hạn như cấu trúc và tính chất của chúng), cho đến liên hệ giữa các đối tượng với nhau, để tránh bỏ sót thông tin quan trọng.


Trong những bài toán phức tạp gồm nhiều công đoạn, các bước trên sẽ phải thực hiện nhiều lần cho mỗi phần của bài toán. Việc tiếp cận có định hướng thế này, ban đầu có thể sẽ hơi tốn thời gian và mệt mỏi trong suy nghĩ, nhưng khi đã thành thạo thì cho thấy hiệu quả lớn, hơn nữa còn rèn luyện được khả năng giải quyết vấn đề một cách độc lập. Người viết bài đã liên tục sử dụng định hướng trên trong việc giảng dạy tại lớp Chuyên đề Toán 9 năm học 2022-2023 và nhận thấy hiệu quả tương đối rõ rệt.

Ví dụ 1. Chứng minh rằng trong 39 số tư nhiên liên tiếp, luôn tìm được một số mà tổng các chứ số của nó chia hết cho 11.


Phân tích. Khi đọc kỹ bài toán, một số câu hỏi sau về các khó khăn là tự nhiên:
a) Tại sao đối tượng được quan tâm là tồng các chữ số ?
b) Dưới điều kiẹn gi thì tồng đó sẽ là bội của 11 ?
c) Tại sao phải cần 39 số tự nhiên liên tiếp? Như thế là ít hay nhiều?
Để đưa được một lập luận trực tiếp nhằm giải quyết các câu hỏi trên, nhìn chung là việc khó hình dung. Các yêu cầu trên có sự liên quan mật thiết với nhau, và hơn nữa tồng các chũ số là một đại lượng không quen thuộc cho lắm, nên một cách tiếp cận khả dũ là việc làm mọi thứ trở nên rõ ràng, từ tính chất của tổng các chữ số hay là quan hệ trong nội bộ của đối tượng, cho đến quan hệ giũa các đối tượng đã xuất hiện.
Một cách tìm hướng giải quyết là đưa ra ví dụ. Khi nhìn vào trường hợp đơn giản nhất cho 39 số tự nhiên liên tiếp chính là các số từ 1 đến 39, chúng ta có thề quan sát được sự biến động của tổng các chũ̃ số và khảo sát được tính chia hết cho 11. Có gì thú vị?

a) Dường như tồng các chư số là tăng dần, nhưng có lúc tổng đó sẽ bị giảm. Vậy khi nào tổng ấy tăng và khi nào tổng ấy giảm ? Quan sát kỹ sẽ thấy rằng: Khi bắt đầu từ số chia hết cho 10 , chẳng hạn là $10 x$ với $x \in \mathbb{Z}^{+}$, thì các số từ $10 x$ đến $10 x+9$ có tổng các chữ số là 10 số tự nhiên liên tiếp. Tổng các chũ số sẽ giảm khi ta “chuyển” tù̀ $10 x+9$ lên $10(x+1)$.
b) Việc chia hết cho 11, nếu nhìn lại ý đầu tiên, thì chúng ta nhận ra rằng vì đã có cách tạo ra 10 giá trị liên tiêp của tổng các chữ số, chỉ cần cố gắng “kéo dài” để tạo ra 11 giá trị liên tiếp của tổng đó thì bài toán sẽ được hoàn tất, bởi trong 11 số tự nhiên liên tiêpp, thế nào cũng có số chia hết cho 11. Do đó việc quan sát vị trí mà tổng các chũ số bị giảm trở nên quan trọng, và đại lượng đó sẽ giảm thế nào ?

  • Có vẻ nhu khi từ $10 x+9$ lên $10(x+1)$ thì tổng các chũ số sẽ giảm 9 đơn vị. Nếu được nhu thê, chúng ta chỉ cần lấy 20 số là $10 x, 10 x+1, \cdots, 10 x+19$ là xong, vì sẽ thu được 11 giá trị liên tiếp cho tổng các chữ sô.
  • Nhưng tại sao bài toán lại cần đến 39 số ? Nếu hình dung một bộ gồm 20 số liên tiếp, bắt đầu từ số chia hết cho 10, là ứng viên tiềm năng để giải quyết bài toán, thì chúng ta không cần đến 39 số để chắc chắn chọn được, mà cần quãng 30 số là đủ. Nghĩa là nhận xét về sự thay đổi được đưa ra phía trên có thể không đúng.
  • Vậy chúng ta tiếp tục kiểm tra khi nào nhận xét “giảm 9 đơn vị” đúng và khi nào điều đó sai, hay có thể tạm gọi là chú ý đến sự xuất hiện của những thứ “ngoài quy luật”. Thử với các giá trị tiếp theo của $x$, rất đáng chú ý khi nhận ra rằng, nhận xét sẽ sai khi có bước chuyển từ 99 lên 100, hay từ 199 lên 200,… Nói cách khác, miễn là $10(x+1)$ không chia hết cho 100 thì nhân xét đúng.

Có thể rút ra được gì từ các nhận định trên?
a) Nếu trong 39 số mà không có số nào chia hết cho 100, thi chọn được bộ 20 số liên tiếp từ $10 x$ đến $10 x+19$, mà có thể hoàn toàn yên tâm về tính “liên tiếp” của tổng các chưu số trong nhũ̃ng số đang được xét, và bài toán sê xong.
b) Lỡ nhu trong 39 số ban đầu, có số chia hết cho 100 thì sao ? Như đã chỉ ra, chúng ta chỉ cần 20 số có dạng $10 x$ đến $10 x+19$, mà trong chúng sẽ không có số nào chia hết cho 100. Có thể hiểu rằng số chia hết cho 100, mà tạm gọi là a, sê “phân đôi” 39 số mà bài toán cho thành 2 phần: một phần gồm các số từ a trở lên, và một phần gồm các số từ a-1 trở xuống. Vì ban đầu chúng ta có 39 số, theo Nguyên lý Dirichlet, phải có một phần được tạo ra gồm ít nhất 20 số.
Và thế là xong. Bây giờ chỉ là sắp xếp và viết lại các nhận định trên thành một lời giải ngắn gọn. Khi viết thành văn thì các suy luận trên có vẻ dài dòng, nhưng trên thực tế khi suy nghĩ, mọi thứ chỉ ở dạng ý tưởng, nên việc triển khai có thề diễn ra rất nhanh.

Chứng minh. Với mỗi số nguyên dương $n$, gọi $S(n)$ là tổng các chữ số của $n$. Trước hết chúng ta chứng minh rằng, với $x$ là số nguyên dương sao cho $100 \nmid 10(x+1)$, có một trong các số $10 x, 10 x+1, \cdots, 10 x+19$ có tổng các chữ số chia hết cho $11 .$. Thật vậy, đặt $S(10 x)=a$ thì với $0 \leq k \leq 9$, ta có $S(10 x+k)=a+k$ và $S(10 x+10+k)=$ $a+1+k$. Do đó tổng các chữ số nhận giá trị trong ${a, a+1, a+2, \cdots, a+10}$, là tập hợp gồm 11 số tự nhiên liên tiếp, và trong tập hợp đó có một giá trị chia hết cho 11 . Quay trở lại bài toán. Gọi 39 số tự nhiên của đề bài lần lượt là là $a, a+1, \cdots, a+38$. Xét các khả năng sau:

  • Trong 39 số này không có số nào là bội của 100. Bởi vì tập hợp ${a, a+1, \cdots, a+9}$ gồm 10 số tự nhiên liên tiếp, trong đó phải có một số chia hết cho 10. Khi đó tồn tại $0 \leq k \leq 9$ để $10 \mid a+k$. Xét các giá trị trong ${S(a+k), S(a+k+1), \cdots, S(a+$ $k+19)}$ thì theo nhận xét ở đầu bài toán, tồn tại một giá trị trong đó là bội của 11.
  • Tồn tại một giá trị $0 \leq k \leq 38$ để $100 \mid a+k$. Khi đó trong các số còn lại, không còn số nào chia hết cho 100 . Có hai khả năng sau:
  • Nếu $k \leq 18$, xét tập hợp ${S(a+k), S(a+k+1), \cdots, S(a+k+19)}$ thì theo nhận xét ở đầu bài toán, tồn tại một giá trị trong đó là bội của 11 .
  • Nếu $k \geq 19$, xét tập hợp ${S(a+k), S(a+k-1), \cdots, S(a+k-19)}$ thì theo nhận xét ở đầu bài toán, tồn tại một giá trị trong đó là bội của 11 .
    Tóm lại, trong 39 số tự nhiên liên tiếp, luôn có số mà tổng các chữ số là bội của 11.

Ví dụ trên cũng cho thấy được một hiện tượng rất thú vị và hầu như luôn đúng, đó là khi quá trình phân tích đủ cẩn thận, việc trình bày lời giải chỉ là một cách sắp xếp và viết ngược lại những ý tưởng chính trong mạch suy luận mà thôi. Để kết thúc bài toán này một cách trọn vẹn, bây giờ là một câu hỏi dành cho các bạn.
Ví dụ 2. Thay vì 39 số, chúng ta chỉ xét 38 số thôi. Liệu bài toán còn đúng không ?
Một gợi ý cho các bạn là hãy đọc lại thật cẩn thận từng bước suy luận, và xem vấn đề diền ra ở đâu. Chú ý rằng nếu như bài toán vần đúng, các bạn phải cung cấp một chứng minh, còn nếu kết quả trở nên sai thì hãy chỉ ra một phản ví dụ. Bây giờ chúng ta đến với một bài toán khác cũng tương đối cổ điển.

Ví dụ 3. Cho sáu số nguyên dương đôi một phân biệt và đều nhỏ hơn 10. Chứng minh rằng luôn tìm được ba số trong đó, mà có một số bằng tồng hai số còn lại.

Phân tích. Một số câu hỏi có thể được đặt ra:
a) Tại sao lại xét 6 số trong ${1,2, \cdots, 9}$ ?
b) Việc có một số bằng tổng hai số còn lại có ý nghĩa gì ? Số nào sẽ bằng tổng của hai số nào ? Khó khăn tại đây đến từ việc chúng ta không xác định được điều trên.
Mà nếu đã không xác định được rõ ràng mọi thứ ngay lập tức, thì tốt nhất là lấy ví dụ cu thể để quan sát thôi. Khi lấy thử một vài ví dụ đề khảo sát, dù có ít bộ ba số hay nhiều bộ ba số thoả mãn yêu cầu bài toán, luôn có một nhận xét quan trọng xuất hiện: tồn tại hai số có tổng bằng số lớn nhất.
Vậy từ đây một hướng đi khả dĩ là tìm hiểu xem số lớn nhất như thế nào, đồng thời làm thế nào có thề viết được số đó thành tổng của hai số tự nhiên phân biệt khác. Khi đã làm được điều đó, hãy xem các thông tin vừa nhận được liên hệ gì với giả thiết ban đầu, mà cụ thể là những số nào xuất hiện trong các cách phân tích thành tổng ấy. Thực ra cũng không quá nhiều trường hợp để giải quyết, vì số lớn nhất thì cũng phải không nhỏ hơn 6.

Chứng minh. Gọi 6 số đã cho là $1 \leq a_1<a_2<\cdots<a_6 \leq 9$. Theo giả thiết trên và đề bài thì $a_k \geq k$ với $1 \leq k \leq 6$. Xét các khả năng sau:

  • Nếu $a_6=9$ thì $1 \leq a_k \leq 8$ với $1 \leq k \leq 5$. Phân các số nguyên dương từ 1 đến 8 thành bốn tập hợp ${1,8},{2,7},{3,6},{4,5}$. Theo nguyên lý Dirichlet, trong các số từ $a_1$ đến $a_5$, có ít nhất hai số thuộc vào cùng một tập hợp. Tổng hai số đó bằng 9 , nên tồn tại $1 \leq i<j \leq 5$ để $a_i+a_j=9$.
  • Nếu $a_6=8$ thì $1 \leq a_k \leq 7$ với $1 \leq k \leq 5$. Phân các số nguyên dương từ 1 đến 8 thành bốn tập hợp ${1,7},{2,6},{3,5},{4}$. Theo nguyên lý Dirichlet, trong các số từ $a_1$ đến $a_5$, có ít nhất hai số thuộc vào cùng một tập hợp. Tổng hai số đó bằng 8 , nên tồn tại $1 \leq i<j \leq 5$ để $a_i+a_j=8$.
  • Nếu $a_6=7$ thì $1 \leq a_k \leq 6$ với $1 \leq k \leq 5$. Phân các số nguyên dương từ 1 đến 7 thành ba tập hợp ${1,6},{2,5},{3,4}$. Theo nguyên lý Dirichlet, trong các số từ $a_1$ đến $a_5$, có ít nhất hai số thuộc vào cùng một tập hợp. Tổng hai số đó bằng 7 , nên tồn tại $1 \leq i<j \leq 5$ để $a_i+a_j=7$.
  • Nếu $a_6=6$ thì $1 \leq a_k \leq 5$ với $1 \leq k \leq 5$. Phân các số nguyên dương từ 1 đến 5 thành ba tập hợp ${1,5},{2,4},{3}$. Theo nguyên lý Dirichlet, trong các số từ
  • $a_1$ đến $a_5$, có ít nhất hai số thuộc vào cùng một tập hợp. Tổng hai số đó bằng 6 , nên tồn tại $1 \leq i<j \leq 5$ để $a_i+a_j=6$.
  • Tóm lại thì luôn có hai số bằng tổng của số lớn nhất. Bài toán kết thúc.

Khai thác thêm bài toán này có thể thấy nhiều điều thú vị sau:
a) Câu hỏi đầu tiên vẫn chưa được giải quyết triệt để khi phân tích. Tuy nhiên, với trường hợp $a_6=9$, nhận thấy rằng việc chọn ra 6 số là để vừa đủ cho việc sử dụng Nguyên lý Dirichlet. Một câu hỏi tự nhiên là nếu bài toán chỉ xét 5 số thay vì 6 số, thì các lập luận sẽ biến đổi thế nào, và liệu kết luận của bài toán còn đúng ?
b) Phát biểu khác đi một chút, liệu số lượng số nhỏ nhất cần chọn để chắc chắn có một số bằng tổng hai số khác, là bao nhiêu? Hơn nữa thay vì giải quyết bài toán như trường hợp ban đầu, khi các số không lớn hơn 9 , điều gì sẽ xảy ra khi thay 9 bởi một số nguyên dương $n$ bất kỳ ? Liệu các câu hỏi tương tự có thể được giải quyết ?
Từ đó có thể thu được bài toán sau, xin dành cho các bạn tự luyện tập.
Ví dụ 4. Cho số nguyên dương $n \geq 3$. Tìm số nguyên dương $k$ nhỏ nhất sao cho với mọi cách chọn ra $k$ số nguyên dương đôi một phân biệt từ tập hợp ${1,2, \cdots, n}$, luôn chọn được ba số trong đó, mà có một số bằng tổng hai số kia.

Ví dụ 5. Với n là số nguyên dương, chọn ra $n+1$ số từ tập hợp ${1,2, \cdots, 2 n}$.
a) Chứng minh rằng có hai số nguyên tố cùng nhau.
b) Chứng minh rằng có hai số mà thương của chúng là số nguyên.

Phân tích. Một số câu hỏi có thể được đặt ra như sau:
a) Tại sao phải cần chọn ra $n+1$ số ?
b) Sự nguyên tố cùng nhau, và việc thương là số nguyên, có ý nghĩa số học gì ? Nếu định nghĩa một cách số học, thì hai số được gọi là nguyên tố cùng nhau khi và chỉ khi chúng không có ước nguyên tố chung. Khi thử tiếp cận theo việc khảo sát các ước nguyên tố của $n+1$ số, mọi chuyện sẽ trở nên rất phức tạp vi chúng ta không biết những số nào được chọn ra, hơn nữa bài toán chỉ yêu cầu một sự tốn tại, nên nếu đi khảo sát toàn bộ cấu trúc của tập hợp ước nguyên tô, thì đó có vè là một yêu cầu quá sức. Hơn nũa, một vấn đề khác làm hướng tiếp cận này trở nên không khả thi, đó là trong bài toán không hề có dấu hiệu gì cho thấy nên tìm hiểu một cách chi tiết về các cấu trúc số $h o c$.
Do đó chúng ta sẽ thử một góc nhìn khác. Tập trung vào câu hỏi đầu tiên, một vấn đề được đặt ra nhu sau: nếu như chỉ lấy $n$ số thì sao? Có thể tìm ngay được phản ví dụ với việc chọn $n$ số chã̃n, thì hai số nào cũng có ước nguyên tố chung là 2. Vậy trong trường hợp tạm gọi là xấu nhất, thế nào cũng có ít nhất một số lẻ. Và liệu số lẻ này có vai trò và quan hệ thế nào với các số chã̃n, khi cần khảo sát sự nguyên tố cùng nhau?
Viết một vài trường hợp nhỏ, chúng ta nhận ra rằng khi số lẻ ấy được kết hợp với số liền trước hay số liền sau, thi sẽ tạo ra một cặp số nguyên tố cùng nhau. Từ đó một câu hỏi

nảy sinh: nếu như chọn $n+1$ số bất kỳ, thi liệu luôn có hai số tự nhiên liên tiếp ? Điều này có thể được kiểm chứng dễ dàng, nên ý đầu tiên của bài toán đến đây là hoàn thành. Sự kiện “chia hết” là một yếu tố khó kiểm soát. Bây giờ chẳng hạn như đã chọn trước một số nguyên dương a, các số chia hết cho a sẽ là ka, hoặc các ước của a thì luôn có dạng $a / k$. Vấn đề là, chúng ta không xác định được khi chọn ra $n+1$ số bất kỳ, sẽ có các số nào liên quan đến a xuất hiện, hơn nữa không chắc chắn việc thương của chúng liệu có phải số nguyên. Vậy thì chúng ta sẽ thử làm mạn đánh giá lên để khử được sự ngẫu nhiên ấy: nếu như chọn ra được một bộ càng nhiều số càng tốt mà liên quan đến a, đồng thời hai số nào trong đó cũng có thương là số nguyên, thì bộ số ấy chỉ nên được chọn tối đa một phần tử nhằm tránh việc chia hết.
Làm rõ ý tưởng này, chúng ta sẽ nhận ra $a, 2 a, 4 a, \cdots$ là lựa chọn tốt nhất có thể nếu xét các số tù̀ a trở lên. Khi chú ý đến các số tù̀ a trở xuống và hiệu chỉnh, lưa chọn phù hợp cho bộ số cần tìm chính là $a, 2 a, 4 a, \cdots$ với a là số lẻ. Có n bộ như thê, và thế là xong.

Chứng minh.
a) Chia tập hợp ${1,2, \cdots, 2 n}$ thành $n$ tập hợp ${2 k-1,2 k}$ với $1 \leq k \leq n$. Vì ban đầu có $n+1$ phần tử được chọn ra, theo Nguyên lý Dirichlet, phải có hai phần tử nào đó thuộc cùng một tập hợp con được nêu ra phía trên. Đây là hai số tự nhiên liên tiếp nên chúng nguyên tố cùng nhau.
b) Với $a$ là số lẻ và $1 \leq a \leq 2 n$, ta định nghĩa
$$
S_a=\left\{x \in \mathbb{Z}^{+}, 1 \leq x \leq 2 n \mid \exists k \in \mathbb{Z}^{+}: x=2^k a\right\}
$$
Mỗi số nguyên dương không vượt quá $2 n$ đều thuộc về một tập hợp $S_a$ nào đó. Có $n$ tập hợp như thế, mà ban đầu có $n+1$ số được chọn, nên Nguyên lý Dirichlet cho thấy rằng phải có hai số cùng nằm trong một tập hợp $S_a$ nào đó. Gọi hai số đó là $2^s a$ và $2^t a$ với $0 \leq s<t$ thì thương của chúng là $2^{t-s} a$, là một số nguyên.

Như thường lệ, bài toán chưa kết thúc ngay tại đây, mà chúng ta đặt ra thêm một vài quan sát nữa. Việc chọn $n+1$ số trong tập hợp ${1,2, \cdots, 2 n}$, như đã phân tích, là vừa đủ để vượt qua ngưỡng “lớn nhất” của sự kiện không có hai số nào nguyên tố cùng nhau. Một lẽ dĩ nhiên là chúng ta muốn xác lập một ngưỡng tương tự cho sự kiện chia hết: liệu có thể chọn được tối đa bao nhiêu sô, mà không có hai số nào có thương là số nguyên ?
Hơn nữa, nếu như kết hợp cả hai vấn đề, nghĩa là có thể chọn được tối đa bao nhiêu số để không có hai số nào nguyên tố cùng nhau và đồng thời không có hai số nào có thương là số nguyên, chúng ta thu được bài toán sau trong đề thi chọn Đội tuyển năm 2017 của Trường Phổ thông Năng khiếu để tham dự Kỳ thi Học sinh giỏi Quốc gia môn Toán bậc THPT.

Ví dụ 6 (PTNK 2017). Xét tập hợp $S={1,2, \cdots, 2017}$. Liệu có thể chọn ra tôi đa bao nhiêu số nguyên dương từ $S$, sao cho không có hai số nào nguyên tố cùng nhau và đồng thời không có hai số nào có thương là số nguyên ?

Theo trí nhớ của người viết bài cũng tham dự kỳ thi năm ấy, không có thí sinh nào giải quyết được bài toán trên. Mặc dù vậy, khi phân tích kỹ, đặc biệt là về sự kiện chia
hết, các bạn có thể tìm được ngay đáp số và thậm chí là một ví dụ thoả mãn yêu cầu bài toán.
Các bài toán trên đều minh hoạ cho một bước chuyển đổi quan trọng từ những phân tích dài dòng bằng chữ thành các suy diễn gãy gọn được diễn đạt bằng ký hiệu. Vì mỗi tình huống mỗi khác, điều quan trọng nhất vẫn là đọc thật kỹ những giả thiết được đưa ra và nắm chắc những yêu cầu cẩn thiết. Một điều tối kỵ là không được bịa ra thêm giả định vô căn cứ để ép vào mạch suy luận. Chúng ta kết thúc bằng một bài toán thú vị, mặc dù trông có vẻ nhiều khó khăn, và phương châm vẫn là… nghĩ đơn giản thôi

Ví dụ 7. Cho các số tự nhiên tù 1 đến 2023. Hỏi có thể chọn ra được nhiều nhất bao nhiêu số sao cho tổng của hai số bất kì trong chúng không chia hết cho hiệu của nó ?
Phân tích. Một số câu hỏi sau được đặt ra khi đọc kỹ đề bài.
a) Giả định chia hết của bài toán rất kỳ quặc. Có cách nào diễn đạt lại mọi thứ cho rồ ràng hơn hay không, và làm sao để khai thác được điều kiện ấy ?
b) Liệu có thể tìm được một ví dụ với tương đối nhiều số ?
Chúng ta tập trung giải quyết yêu cầu đầu tiên. Viết rõ lại bằng ký hiệu, đó là với $a>b$ là hai số nguyên dương phân biệt được chọn, ta phải có $a-b \nmid a+b$. Vì các số này được chọn bất kỳ và các biểu thức xuất hiện đẹ̀u là bậc nhất, việc tìm kiếm một quan hệ số học giũ̃a a và b chỉ bằng giả định trên là không khả thi. Nếu không tin, các bạn có thể thư!
Xoay sang câu hỏi thứ nhì. Thử tiếp cận vấn đề một cách tương đối ngây thơ như sau: cứ lần lượt bắt đầu tù số 1, liệu có thề lấy được những số nào tiểp theo? Dĩ nhiên không phải lúc nào việc xử lý vấn đè̀ theo cách tham lam cũng cho một kết quả tối u’u, nhung ít nhất vẫn có thêm định hướng và một vài quan sát hữu ích để hiệu chỉnh khi cần thiết.

  • Không lấy được số 2 và số 3, vì ảnh hưởng của số 1 .
  • Lấy được số 4. Cũng bởi thế mà không lấy được số 5 và số 6 .
  • Lấy được số 7, rồi lại bỏ qua số 8 và số 9. Cứ như thế…

Một quan sát về các số được thu nhận cho thấy chúng phải cách nhau ít nhất 3 đơn vị. Liệu điều này có luôn đúng ? Có thề quay về giả định của bài toán để kiểm tra.
Mọi thứ quy về việc chọn ra càng nhiều số càng tốt, mà hai số bất kỳ có hiệu từ 3 trở lên. Để chọn được nhiều số nhất, một lê dĩ nhiên là phải khởi đầu từ số nhỏ nhất, và các khoảng cách giữa các số cũng phải nhỏ nhất có thể. Bây giờ chỉ là xếp lại thành lời giải, và nhớ rằng vì đây là bài toán cực trị, hãy chỉ ra ví dụ.

Chứng minh. Gọi các số được chọn là $1 \leq a_1<a_2<\cdots<a_k \leq 2023$. Trước hết, ta chứng minh rằng với $1 \leq i \leq k-1$ thì $a_{i+1}-a_i \geq 3$. Thật vậy:

Nếu có chỉ số $1 \leq i \leq k-1$ để $a_{i+1}-a_i=1$ thì $a_{i+1}-a_i \mid a_{i+1}+a_i$, mâu thuẫn.

Nếu có chỉ số $1 \leq i \leq k-1$ để $a_{i+1}-a_i=2$ thì chú ý rằng $a_{i+1}+a_i=2 a_i+2$, ta cũng thu được $a_{i+1}-a_i \mid a_{i+1}+a_i$, lại là một mâu thuẫn.
Do đó nhận xét được chứng minh. Từ đó thì
$$
2023 \geq a_k \geq a_{k-1}+3 \geq a_{k-2}+3 \cdot 2 \geq \cdots \geq a_1+3(k-1) \geq 1+3(k-1)
$$
hay là $2022 \geq 3(k-1)$. Điều này cho thấy $k \leq 675$. Để chọn được 675 số thoả mãn yêu cầu bài toán, với $1 \leq i \leq 675$, chọn $a_i=3 i-2$. Thật vậy, với $1 \leq i<j \leq 675$ thì:

  • $a_j-a_i=3(j-i)$ là một bội của 3 ,
  • $a_j+a_i=3(j+i)-4$ không là một bội của 3 , nên ta luôn có $a_j-a_i \nmid a_j+a_i$. Vậy có thể chọn được tối đa 675 số nguyên dương đôi một phân biệt không vượt quá 2023 mà không có tổng hai số nào chia hết cho hiệu của chúng.

Hi vọng rằng những trình bày phía trên có thể giúp các bạn phần nào đó tự tin và vững vàng hơn trong việc suy luận để giải toán. Dưới đây là một số bài toán để luyện tập.

Bài tập rèn luyện.

Bài 1. Xét bảng ô vuông $10 \times 10$. Mỗi ô vuông của bảng được điền một số nguyên tuỳ ý sao cho hiệu hai số được điền ở hai ô chung một cạnh bất kì đều không vượt quá 1 . Chứng minh rằng tồn tại một số nguyên xuất hiện trên bảng ít nhất 6 lần.

Bài 2. Cho $A B C$ là một tam giác tuỳ ý. Mỗi điểm trên mặt phẳng được tô bởi một trong hai màu xanh hoặc đỏ. Chứng minh rằng tồn tại hai điểm màu đỏ có khoảng cách bằng 1, hoặc tồn tại một tam giác có ba đỉnh màu xanh mà đồng dạng với tam giác $A B C$.
Bài 3. Có 20 viên bi được xếp thành một hàng ngang trên bàn, trong đó có 10 viên bi màu xanh và 10 viên bi màu đỏ. Chứng minh rằng có thể chọn ra một bộ gồm 10 viên bi liên tiếp mà trong đó số viên bi màu xanh bằng số viên bi màu đỏ.

Bài 4. Cho $A={1,2,3, \cdots, 100}$. Lấy $S$ là tập hợp con của $A$ sao cho các tồng hai phần tử phân biệt bất kỳ của $S$ thì có các số du đôi một phân biệt khi chia cho 100. Chứng minh rằng $S$ có không quá 14 phần tử, và chỉ ra một tập hợp $S$ có 10 phần tử.
Bài 5. Có một bộ các quả cân có tính chất sau:
i) Trong bộ có ít nhất 5 quả cân có trọng lượng khác nhau.
ii) Với hai quả cân bất kỳ, tìm được hai quả cân khác có tồng trọng lượng bằng với tổng trọng lượng của hai quả cân đó.
Bộ quả cân này có ít nhất là bao nhiêu quả cân?
Bài 6. Chọn ra $k$ số nguyên dương phân biệt là ước của $6^{2023}$.
a) Chứng minh rằng nếu $k=5$ thì tồn tại hai số có tích là số chính phương.
b) Chứng minh rằng nếu $k=21$ thì tồn tại sáu số có tích là một luỹ thừa bậc 6.

Bài 7. Cho số nguyên dương $n \geq 2$. Chứng minh rằng khi chọn ra $n+2$ số nguyên dương từ tập hợp $S={1,2, \cdots, 3 n}$, luôn tồn tại hai số $x, y$ đề $n<x-y<2 n$.

Bài 8. Cho tập hợp $S={1,2, \cdots, 2023}$. Xét tập hợp con $T \subseteq S$. Nếu $T$ không chứa hai phần tử nào có hiệu trong $E$ thì có tối đa bao nhiêu phần tứ, với:
a) $E={3 ; 6 ; 9}$
b) $E={4 ; 7}$
Bài 9. Lớp $9 A$ có 6 học sinh tham gia kỳ thi chọn đội tuyển môn Toán, và nhận được 6 điểm số khác nhau là các số tự nhiên không vượt quá 20. Gọi m là trung bình cộng các điểm số của 6 học sinh trên. Hai học sinh được gọi là lập thành một cạ̣p hoàn hảo nếu như trung bình cộng điểm số của hai em đó lớn hơn $m$.

a) Chứng minh rằng không thề chia 6 học sinh thành 3 cặp mà mỗi cặp đều hoàn hảo.
b) Trong 6 học sinh trên, có thể có nhiều nhất bao nhiêu cặp hoàn hảo ?
Bài 10. Có 8 kì thủ thi đấu giải cờ vua Candidates 2023 theo thể thức vòng tròn một lượt. Tại mỗi trận đấu phân định thắng thua, người thắng được 1 điểm còn người thua được 0 điểm; tại mỗi trận hòa thì mỗi người được 0.5 điểm.
a) Chứng minh rằng sau 3 vòng đầu tiên, luôn tìm được hai người có số điểm bằng nhau.
b) Giả sử rằng sau khi kết thúc giải, tất cả các kì thủ đều có số điểm khác nhau. Tìm số điểm ít nhất có thể của người chiến thắng.
c) Giải lại bài toán khi giải đấu diễn ra theo thể thức vòng tròn hai lượt.

Cực trị hình học (Lớp 9)

Bài toán cực trị hình học thường xuất hiện trong các kì thi học sinh giỏi cũng như thi tuyển sinh, đây là câu hỏi gây khó khăn cho nhiều bạn học sinh vì để giải bài toán cực trị đòi hỏi các kiến thức tổng hợp: bài toán quỹ tích, sử dụng các bất đẳng thức đại số,… ngoài ra cũng phải biết và vận dụng được một số bài toán cực trị cơ bản. Bài viết này giúp các em làm quen với các bài toán cực trị trong chương trình lớp 9, từ đó giúp ôn tập tốt hơn trong kì thi tuyển sinh sắp tới.

Cực trị hình học là các bài toán tìm giá trị lớn nhất, giá trị nhỏ nhất của các đối tượng hình học như: các biểu thức về độ dài, diện tích, chu vi,…khi giá trị của các biểu thức này thay đổi.

Ta có một số chú ý sau khi giải bài toán cực trị hình học.

Chú ý 1. Để tìm giá trị lớn nhất của biểu thức $P$. Ta thường làm theo các bước sau:

  • Chứng minh $P \leq M$ ( $M$ phải là giá trị không đổi).
  • Tìm điều kiện để xảy ra đẳng thức.
  • Kết luận.

Chú ý 2. Để chứng minh với mô hình $H$ có biểu thức $P$ đạt giá trị lớn nhất (hoặc nhỏ nhất), ta có thể chọn mô hình $H^{\prime}$ bất kì với biểu thức tương ứng là $P^{\prime}$ và ta chứng minh $P \geq P^{\prime}$ (hoặc $P \leq P^{\prime}$ ).

Chú ý 3. Để làm các bài toán cực trị hay bất đẳng thức thường có hai hướng để suy nghĩ:

  • Đưa bài toán ban đầu về các bài toán cực trị quen thuộc đã biết cách giải.
  • Sử dụng các bất đẳng thức Đại số áp dụng lên các yếu tố Hình học.

Một số bài toán cực trị quan trọng.

Tính chất 1. (Đường xiên và hình chiếu) Cho điểm $A$ và đường thẳng $d, M$ là điểm thay đổi trên $d$. Khi đó, $A M$ nhỏ nhất khi và chỉ khi $M$ là hình chiếu vuông góc của $A$ trên $d$.

Tính chất 2. (Bất đẳng thức tam giác) Cho 3 điểm $A, B, C$.

  • $A B+B C \geq A C$. Đẳng thức xảy ra khi và chỉ khi $B$ nằm giữa $A$ và $C$.
  • $|A C-A B| \leq B C$. Đẳng thức xảy ra khi $A, B, C$ thẳng hàng và $A$ nằm ngoài đoạn thẳng $B C$.

Tính chất 3. Trong một tam giác vuông thì độ dài đuờng cao xuất phát tù đỉnh góc vuông không lớn hơn nủa độ dài canh huyền.
Chứng minh
Cho tam giác $A B C$ vuông tại $A$, đường cao $A H$. Cần chứng minh $A H \leq \frac{1}{2} B C$.
Gọi $M$ là trung điểm của $B C$ ta có $A M=\frac{1}{2} B C$.
Mà $A H \leq A M$. Suy ra $A H \leq \frac{1}{2} B C$.
Đẳng thức xảy ra khi $H \equiv M$ hay tam giác $A B C$ vuông cân.

Tính chất 4. Cho đường tròn $(O)$ và dây cung $B C$ cố định. Tìm điểm $A$ thuộc cung lớn $\overparen{B C}$ sao cho
a) Chu vi tam giác ABC lớn nhất.
b) Diện tích tam giác ABC lớn nhất.
Chứng minh
a) Trên tia đối của tia $A B$ lấy điểm $D$ sao cho $A D=A C \Rightarrow A B+A C=B D$. Hơn nữa $\angle B D C=\frac{1}{2} \angle B A C$ không đổi.
Suy ra $D$ thuộc cung chứa góc $\frac{1}{2} \angle B A C$ dựng trên đoạn $B C$.
Do đó $B D$ lớn nhất khi $B D$ là đường kính, lúc này $A$ là điểm chính giữa $\overparen{\mathrm{BC}}$.
Vậy chu vi tam giác $A B C$ lớn nhất $\Leftrightarrow A$ là điểm chính giữa cung $B C$.
b) Vẽ đường cao $A H$, gọi $M$ là trung điểm $B C$.
Ta có $A H \leq A M \leq O A+O M$ không đổi.
Diện tích tam giác $A B C$ lớn nhất khi và chỉ khi $A H$ lớn nhất hay khi $H \equiv M$.
Lúc này $A$ là điểm chính giữa cung $B C$.
Vậy diện tích tam giác $A B C$ lớn nhất $\Leftrightarrow A$ là điểm chính giữa $\overparen{\mathrm{BC}}$.

Tính chất 5. Cho đường tròn $(O)$ và điểm $A$ nằm ngoài đường tròn. Tìm $M$ thuộc (O) đề AM là lớn nhất, nhỏ nhất.
Chứng minh.
a) Ta có $A M \leq O A+O M$. Đẳng thức xảy ra khi $O$ nằm giữa $A, M$. Vậy $A M$ lớn nhất khi và chỉ khi $M$ là giao điểm của tia đối tia $O A$ và $(O)$.
b) Tương tự như trên ta có $A M \geq O A-O M$. Đẳng thức xảy ra khi $M$ nằm giữa $O$ và $A$.
Vậy $A M$ nhỏ nhất khi và chỉ khi $M$ là giao điểm của tia $O A$ và $(O)$.

Bất đẳng thức thường dùng. Cho các số $a, b, c$ không âm. Ta có các bất đẳng thức sau:

  • $a+b \geq 2 \sqrt{a b}$
  • $a^2+b^2 \geq \frac{1}{2}(a+b)^2 \geq 2 a b$.
  • $a+b+c \geq 3 \sqrt[3]{a b c}$
    $\frac{1}{a}+\frac{1}{b} \geq \frac{4}{a+b}$
  • $a+b \leqslant \sqrt{2\left(a^2+b^2\right)}$.
    Dấu bằng xảy ra khi và chỉ khi $a=b$.

Một số ví dụ

Ví dụ 1. Cho tam giác $A B C$ có $\angle B A C=60^{\circ}$. M là điểm thay đổi trên cạnh $B C$.
Gọi $D$, E lần lượt là hình chiếu của $M$ trên $A B, A C$. Tìm vị trí của $M$ đề $D E$ có độ dài nhỏ nhất.
Lời giải.
Gọi $I$ là trung điểm $A M$.
Ta có $A D M E$ nội tiếp đường tròn $(I)$. Kẻ đường kính $D F$ của đường tròn $(I)$.
Xét tam giác $D F E$ vuông tại $E$.
Ta có $\angle D F E=\angle A D E=60^{\circ}($ cùng chắn $\overparen{\mathrm{DE}}$ ) $\Rightarrow \angle F D E=30^{\circ}$.
Suy ra $D E=D F \cos \widehat{D F E}=\frac{D F}{2}=\frac{A M}{2}$.
Do đó $D E$ nhỏ nhất khi và chỉ khi $A M$ nhỏ nhất, hay $M$ là chân đường cao hạ từ $A$. Vậy $D E$ nhỏ nhất khi và chỉ khi $M$ là chân đường cao từ $A$ của tam giác $A B C$.

Ví dụ 2. Cho đuờng tròn $(O)$ và dây cung $B C$ cố định. A là điểm thay đổi trên cung lơn BC. Gọi I là tâm đường tròn nội tiếp của tam giác $A B C$.
a) Tìm vị trí của A để diện tích tam giác BIC là lớn nhất.
b) Tìm vị trí của A để AI lớn nhất.
Lời giải.
a) Ta có $\angle B A C \Rightarrow \angle B I C=90^{\circ}+\frac{1}{2} \angle A$ không đổi. Do đó $I$ thuộc cung chứa góc $\alpha=90^{\circ}+\frac{1}{2} \angle A$ dựng trên đoạn $B C$.
Khi đó diện tích tam giác $I B C$ lớn nhất khi và chỉ $I$ là điểm chính giữa cung, hay $A$ là điểm chính giữa cung $B C$.
b) $A I$ cắt $(O)$ tại $D$ khác $A, D$ là điểm chính giữa cung $B C$. Ta có $D I=D C$ không đổi.
Ta có $A I=D A-D I$, do đó $A I$ lớn nhất khi và chỉ khi $D A$ lớn nhất, hay $D A$ là đường kính, khi đó $A$ là điểm chính giữa cung $B C$.
Vậy $A I$ lớn nhất khi và chỉ khi $A$ là điểm chính giữa cung $B C$.

Ví dụ 3. Cho tam giác $A B C$ nội tiếp đường tròn w. P là một điểm thay đổi thuộc cung BC không chúa A. Gọi $H, K$ lần lượt là hình chiếu của A trên $P B, P C$. Tìm vi trí của $P$ để
a) Độ dài đoạn thẳng HK là lớn nhất.
b) Giá trị biểu thúc $A H \cdot P B+A K \cdot P C$ là lớn nhất.
Lời giải.
a) Ta có $\triangle A H B \backsim \triangle A K C \Rightarrow \triangle A H K \sim \triangle A B C$.
Suy ra $\frac{H K}{B C}=\frac{A H}{A B} \leqslant 1$. Do đó $K H \leqslant B C$.
Đẳng thức xảy ra khi $H \equiv B$ hay $A P$ là đường kính.
Vậy $K H$ lớn nhất bằng $B C$ khi $A P$ là đường kính.
b)
$$
\text { Ta có: } \begin{aligned}
A H \cdot P B+A K \cdot P C & =2 S_{A P B}+2 S_{A P C} \
& =2 S_{A B P C} \
& =2\left(S_{A B C}+S_{P B C}\right)
\end{aligned}
$$
Suy ra $A H \cdot P B+A K \cdot P C$ lớn nhất khi và chỉ khi $S_{P B C}$ lớn nhất, hay $P$ là điểm chính giữa cung $B C$.
Vậy $A H \cdot P B=A K \cdot P C$ lớn nhất khi $P$ là điểm chính giữa cung $B C$.

Ví dụ 4. (Thi vào lớp 10 Chuyên Toán trường Chuyên Lam Sơn tỉnh Thanh Hóa năm 2010) Cho đường tròn $(O)$ bán kính $R=1$ và điểm $A$ thỏa $O A=\sqrt{2}$. Từ $A$ vẽ các tiếp tuyến $A B, A C$ với $B, C$ là các tiếp điểm. Các điểm $D, E$ thay đổi trên các đoạn $A B, A C$ sao cho $\angle D O E=45^{\circ}$.
(a) Chứng minh $D E$ tiếp xúc với $(O)$.
(b) Tìm giá trị lớn nhất và nhỏ nhất của $DE$.
Hướng dẫn giải
(a) Ta chứng minh được $A B O C$ là hình vuông. Đường thẳng qua $O$ vuông góc $O D$ cắt $A C$ tại $F$, suy ra $\angle D O E=\angle F O E$.
Ta có $\triangle O B D=\triangle O C F \Rightarrow C F=B D, O F=O D$.
Khi đó $\triangle O E F=\triangle O E D \Rightarrow \angle O E F=\angle O E D$, vẽ $O H \perp D E$, suy ra $O H=O C$, do đó $D E$ là tiếp tuyến của $(O)$.
(b) Ta có $E H=C E, B D=D B$, suy ra $A E+A D+D E=A B+A C=2$.
Đặt $x=A D, y=A E$, suy ra $D E=\sqrt{x^2+y^2}$ và $x+y+\sqrt{x^2+y^2}=2$.
Ta có $\sqrt{x^2+y^2} \leq x+y \leq \sqrt{2\left(x^2+y^2\right)}$, suy ra $2 \sqrt{x^2+y^2} \leq x+y+\sqrt{x^2+y^2} \leq(1+$ $\sqrt{2}) \sqrt{x^2+y^2}$, từ đó suy ra $2-\sqrt{2} \leq \sqrt{x^2+y^2} \leq 1$ hay $2-\sqrt{2} \leq D E \leq 1$.

Từ đó $DE$ lớn nhất bằng 1 khi D trùng B, nhỏ nhất là $2 – \sqrt{2}$ khi $AD = AE$.

Ví dụ 5. Cho nửa đường tròn đường kính $BC=2a$, $A$ thay đổi trên nửa đường tròn. Đường cao $AH$.

a) Tìm giá trị lớn nhất của $BH + AH$.

b) Phân giác góc $BAH, CAH$ cắt $BC$ tại $MN$. Tìm vị giá trị lớn nhất của $MN$.

Lời giải.

a) Rõ ràng $BH + AH$ lớn nhất chỉ khi $H$ thuộc đoạn $OC$ vì nếu $H$ thuộc đoạn $BC$ ta lấy $A’$ đối xứng với $A$ qua trung trực $BC$ ta sẽ có $A’H + BH’ > AH+BH$.

Khi đó $BH + AH = BO + OH + AH$ = a + OH + AH$.

Mà $OH + AH \leq \sqrt{2(OH^2+AH^2)} = a\sqrt{2}$

Do đó $AH + BH \leq a + a\sqrt{2}$, đẳng thức xảy ra khi $AH = OH$ và $H$ là trung điểm $OC$.

Vậy giá trị lớn nhất của $BH+AH$ là $a+a\sqrt{2}$ khi $H$ là trung điểm $OC$.

b) Ta có $\angle BAN = \angle BAH + \angle HAN = \angle ACB + \angle CAN = \angle BNA$, suy ra $BN = BA$

Chứng minh tương tự thì $CM = AC$

Khi đó $MN = BN +CM – BC = AB + AC – BC \leq \sqrt{2{AB^2+AC^2}} – BC = 2a(\sqrt{2}-1)$.

Do đó $MN$ lớn nhất là $2a(\sqrt{2}-1)$ khi $AB = AC$.

Bài tập rèn luyện

Bài 1. Cho tam giác $A B C$ nội tiếp đường tròn $(O), A B<A C$. Phân giác trong $\angle B A C$ cắt $(O)$ tại $D$ khác $A$. Trên tia $A B$ lấy $M$ tuỳ ý sao cho đường tròn ngoại tiếp $\triangle A D M$ cắt $A C$ tại $N$ khác $A, C$. Xác định vị trí tâm $I$ của đường tròn ngoại tiếp $\triangle A D M$ để độ dài đoạn thẳng $M N$ nhỏ nhất.

Bài 2. Cho đường tròn tâm $O$ đường kính $B C, A$ là điểm di động trên đường tròn $(O)$ ( $A$ khác $B, C)$. Kẻ $A H \perp B C$ tại $H$. Kẻ $H P \perp A B$ tại $P$. Tìm vị trí điểm $A$ sao cho bán kính đường tròn ngoại tiếp $\triangle B P C$ đạt giá trị lớn nhất.
Bài 3. Cho $\triangle A B C$ vuông tại $A$ có $A B<A C$ ngoại tiếp đường tròn $(O)$.
Gọi $D, E, F$ lần lượt là tiếp điểm của $(O)$ với các cạnh $A B, A C, B C$. $M$ là điểm di động trên đoạn $C E$. Gọi $N$ là giao điểm của $B M$ với cung nhỏ $E F$ của $(O)$. Các điểm $P, Q$ lần lượt là hình chiếu của $N$ trên các đường thẳng $D E, D F$. Xác định vị trí điểm $M$ để độ dài $P Q$ lớn nhất.

Bài 4. Cho 3 đường tròn có tâm thẳng hàng và ngoài nhau, đường tròn thứ tư tiếp xúc ngoài với cả ba đường tròn trên. Chứng minh rằng bán kính đường tròn thứ tư lớn hơn bán kính của một trong ba đường tròn kia.

Bài 5. (Đề thi Olympic 30-4 năm 2000)Trên đường tròn tâm $O$ bán kính $R$ cho năm điểm phân biệt $A, B, C, D, E$ theo thứ tự đó sao cho $A B=B C=D E=R$. Gọi $M, N$ lần lượt là trung điểm của $C D$ và $A E$. Hãy xác định giá trị lớn nhất có thể có của chu vi tam giác $B M N$.

Sử dụng đánh giá bất đẳng thức để giải hệ phương trình

Một trong các phương pháp khác đặc biệt để giải các hệ phương trình là sử dụng bất đẳng thức, kiểu $A \geq 0$, khi đó $A = 0$ chỉ tại các dấu $=$ xảy ra, hoặc $x \geq y \geq z \geq x$, do đó hệ có nghiệm chỉ khi các dấu $=$ đồng thời xảy ra.

Ta cùng tìm hiểu phương pháp này thông qua một số ví dụ, từ đó rút ra kinh nghiệm giải các hệ phương trình khác.

Ví dụ 1. Giải hệ phương trình

$\left\{\begin{array}{l}
x+\frac{2 x y}{\sqrt[3]{x^2-2 x+9}}=x^2+y \\\\
y+\frac{2 x y}{\sqrt[3]{y^2-2 y+9}}=y^2+x
\end{array}\right.$

Lời giải.

$$
\left\{\begin{array}{l}
x+\frac{2 x y}{\sqrt[3]{x^2-2 x+9}}=x^2+y(1) \\\\
y+\frac{2 x y}{\sqrt[3]{y^2-2 y+9}}=y^2+x(2)
\end{array}\right.
$$
Ta có: $x^2-2 x+9 \geq 8 \Rightarrow \sqrt[3]{x^2-2 x+9} \geq 2 \Rightarrow \frac{2 x y}{\sqrt[3]{x^2-2 x+9}} \leq x y$ Tương tự: $\frac{2 x y}{\sqrt[3]{y^2-2 y+9}} \leq x y$
Do đó: $x+\frac{2 x y}{\sqrt[3]{x^2-2 x+9}}+y+\frac{2 x y}{\sqrt[3]{y^2-2 y+9}} \leq x+y+2 x y \leq x+y+x^2+y^2$
(Dấu “=” xảy ra khi và chỉ khi $x=y$ Từ $(1),(2)$ và $(3)$ suy ra $x=y$ Thay $x=y$ vào (1) ta được:
(4) $\Leftrightarrow \sqrt[3]{x^2-2 x+9}=2 \Leftrightarrow x^2-2 x+9=8 \Leftrightarrow(x-1)^2=0 \Leftrightarrow x=1 \Rightarrow$ $y=1$
Vậy nghiệm của hệ phương trình là $(x, y) \in{(0,0) ;(1,1)}$.

Ví dụ 2. (Hệ hoán vị vòng quanh) Giải hệ phương trình

$$\left\{\begin{array}{l}
x^3+3 x^2+2 x-5=y \\\\
y^3+3 y^2+2 y-5=z \\\\
z^3+3 z^2+2 z-5=x
\end{array}\right.$$

Lời giải. Do vai trò bình đẳng trong hoán vị vòng quanh của $x, y, z$ trong hệ trên, ta có thể giả sử
$$
\begin{aligned}
& x=\max {x ; y ; z} \text {. } \\\\
& \text { Vì } y \leq x \text { nên } x^3+3 x^2+2 x-5 \leq x \\\\
& \Leftrightarrow x^3+3 x^2+x-5 \leq 0 \\\\
& \Leftrightarrow(x-1)\left(x^2+4 x+5\right) \leq 0 \text {. } \\\\
& \text { Vì } x^2+4 x+5=(x+2)^2+1>0 \text { nên } x \leq 1 \text {. } \\\\
& \text { Mà } z \leq x \text { nên } z \leq 1 \text {. } \\\\
& \text { Lập luận ngược lại quá trình trên ta được } \\\\
& (z-1)\left(z^2+4 z+5\right) \leq 0 \\\\
& \Leftrightarrow z^3+3 z^2+2 z-5 \leq z \Leftrightarrow x \leq z \text {. } \\\\
& \text { Do đó } x=z \text {. } \\\\
& \text { Suy ra } x=y=z \text {. } \\\\
& \text { Từ đó ta được phương trình } \\\\
& \quad x^3+3 x^2+2 x-5=x \\\\
& \Leftrightarrow(x-1)\left(x^2+4 x+5\right)=0 \Leftrightarrow x=1 \text {. }
\end{aligned}
$$

Ví dụ 3 (Chuyên Toán PTNK 1997) Tìm tất cả các số dương $x, y, z$ thỏa : $\left\{\begin{array}{l}
\frac{1}{x}+\frac{4}{y}+\frac{9}{z}=3 \\\\
x+y+z \leq 12
\end{array}\right.$

Lời giải.

\begin{aligned}
& \text { Ta có }(x+y+z)\left(\frac{1}{x}+\frac{4}{y}+\frac{9}{z}\right) \leq 36 \Leftrightarrow \frac{y}{x}+\frac{4 x}{y}+\frac{z}{x}+\frac{9 x}{z}+\frac{4 z}{y}+\frac{9 y}{z}-22 \leq 0 \Leftrightarrow \\\\
& \frac{(y-2 x)^2}{x y}+\frac{(z-3 x)^2}{x z}+\frac{(3 y-2 z)^2}{y z} \leq 0 \Leftrightarrow y=2 x, z=2 x, 3 y=2 z \text { Từ đó ta } \\\\
& \text { có } x=2, y=4, z=6
\end{aligned}

Ví dụ 4. (PTNK Chuyên Toán 2103) Giải hệ phương trình $\left\{\begin{array}{l}
3 x^2+2 y+1=2 z(x+2) \\\\
3 y^2+2 z+1=2 x(y+2) \\\\
3 z^2+2 x+1=2 y(z+2)
\end{array}\right.$

Lời giải. Cộng ba phương trình lại ta có:
$3\left(x^2+y^2+z^2\right)+2(x+y+z)+3=2(x y+y z+z x)+4(x+y+z) $

$ \Leftrightarrow 3\left(x^2+y^2+z^2\right)-2(x y+y z+x z)-2(x+y+z)+3=0 $
$\Leftrightarrow(x-y)^2+(y-z)^2+(z-x)^2+(x-1)^2+(y-1)^2+(z-1)^2=0 $
$\Leftrightarrow\left\{\begin{array}{l}
x=1 \\\\
y=1 \\\\
z=1
\end{array}\right.
$
Thử lại thấy $(1,1,1)$ là nghiệm của hệ.

Bài tập rèn luyện

Bài 1. Giải hệ phương trình

$\left\{\begin{array}{l}
x+\dfrac{2 x y}{\sqrt{x^2-2 x+5}}=x^2+y \\\\
y+\dfrac{2 x y}{\sqrt{y^2-2 y+5}}=y^2+x
\end{array}\right.$

Bài 2. Giải hệ phương trình $\left\{\begin{array}{l}
y^{3}-6 x^{2}+12 x-8=0 \\\\
z^{3}-6 y^{2}+12 y-8=0 \\\\
x^{3}-6 z^{2}+12 z-8=0
\end{array}\right.$

Bài 3. Tìm các số không âm $x, y, z$ thỏa
$$
\left\{\begin{aligned}
x y z & =1 \\\\
x^3+y^3+z^3 & =x+y+z
\end{aligned}\right.
$$

Định lý Viete và áp dụng

Định lý 1. (Định lý Viete thuận) Cho phương trình bậc hai $a x^2+b x+c=0$ (a,b, c là các hệ số). Nếu phương trình có nghiệm $x_1, x_2$ thì
$$
S=x_1+x_2=\frac{-b}{a}, \text { và } P=x_1 x_2=\frac{c}{a}
$$
Định lý 2. (Định lý Viete đảo) Nếu có hai số $a, b$ thỏa $a+b=S, a b=P$ thì $a, b$ là nghiệm của phương trình
$$
x^2-S x+P=0
$$

Chú ý: Điều kiện để áp dụng định lý Viete là phương trình bậc hai phải có nghiệm, tức là $\Delta \geq 0$.

Ví dụ 1. Cho phương trình $x^2-2(m+1) x+m=0$
(a) Chứng minh rằng phương trình luôn có 2 nghiệm phân biệt $x_1, x_2$
(b) Tính giá trị các biểu thức sau theo $m$
$$
A=x_1^2+x_2^2+x_1+x_2
$$
(c) Tìm $m$ để $A=18$.
Lời giải. $a=1, b=-2(m+1), b^{\prime}=-(m+1), c=m$
a) Ta có $\Delta^{\prime}=b^{\prime 2}-a c=(-m-1)^2-1 \cdot m=m^2+m+1$.

$\Delta=m^2+m+1=m^2+2 \cdot m \cdot \frac{1}{2}+\frac{1}{4}+\frac{3}{4}=$ $\left(m+\frac{1}{2}\right)^2+\frac{3}{4}>0$ với mọi $m$. Vậy phương trình luôn có hai nghiệm phân biệt $x_1, x_2$.
b) Ta có $A=x_1^2+x_2^2+x_1+x_2$
$=\left(x_1+x_2\right)^2-2 x_1 x_2+x_1+x_2$
$=4(m+1)^2-2 m+2(m+1)$
$=4 m^2+8 m+6$.

c) $A=18 \Leftrightarrow 4 m^2+8 m-12=0 \Leftrightarrow m=$ $1, m=-3$.
Vậy $m$ cần tìm là 1 và -3 .

Ví dụ 2. Tìm $m$ để phương trình $x^2-2(m+1) x+m^2-3=0$ có hai nghiệm phân biệt $x_1, x_2$ thỏa $x_1^2+x_2^2+x_1 x_2=$ $m+7$
Lời giải. $a=1, b=-2 m-2, c=m^2-3$.

Ta có $\Delta^{\prime}=b^2-a c=(m+1)^2-\left(m^2-3\right)=2 m+4$. Phương trình có hai nghiệm phân biệt khi và chỉ khi $\Delta^{\prime}=2 m+4>0 \Leftrightarrow m>-2$.

Theo dịnh lý Viete ta có $x_1+x_2=2(m+1), x_1 x_2=$ $m^2-3$

$x_1^2+x_2^2+x_1 x_2=m+7 \Leftrightarrow\left(x_1+x_2\right)^2-x_1 x_2=m+7$ $\Leftrightarrow 4(m+1)^2-\left(m^2-3\right)=m+7 \Leftrightarrow 3 m^2+7 m=0 \Leftrightarrow$ $m=0(n), m=\frac{-7}{3}(l)$.

Vậy giá trị cần tìm của $m$ là $m=0$.

Ví dụ 3. Cho phương trình $x^2-4 m x+3 m^2+1=0$.
a) Tìm $m$ để phương trình có nghiệm.
b) Gọi $x_1, x_2$ là nghiệm của phương trình, tìm hệ thức độc lập $m$ liên hệ giữa $x_1$ và $x_2$.
Lời giải
a) Ta có $\Delta^{\prime}=4 m^2-\left(3 m^2+1\right)=m^2-1$. Phương trình có nghiệm khi và chỉ khi $\Delta^{\prime} \geq 0 \Leftrightarrow m^2-$ $1 \geq 0 \Leftrightarrow m \leq-1$ hoặc $m \geq 1$.
b) Với điều kiện của a) theo định lý Viete ta có $S=x_1+$ $x_2=4 m(1), P=x_1 x_2=3 m^2+1(2)$.
Từ (1), suy ra $m=\frac{1}{4} S$, thế vào (2) ta có $P=3 m^2+1=$ $\frac{3}{16} S^2+1$.
Hay $x_1 x_2=\frac{3}{16}\left(x_1+x_2\right)^2+1$ là hệ thực liên hệ giữa $x_1, x_2$ độc lập với $m$.

Ví dụ 4. Cho phương trình $x^2-2 m x-2 m-3=0$. Chứng minh rằng phương trình luôn có hai nghiệm phân biệt $x_1, x_2$ và tìm giá trị nhỏ nhất của biểu thức $A=x_1^2+x_2^2-$ $x_1 x_2$.
Lời giải

Ta có $\Delta^{\prime}=m^2+2 m+3$.
Vì $m^2+2 m+3=(m+1)^2+2>0 \forall m$ nên $\Delta^{\prime}>0 \forall m$. Vậy phương trình luôn có hai nghiệm phân biệt với mọi $m$.

Theo định lý Viete ta có $x_1+x_2=2 m, x_1 x_2=-2 m-3$. Khi đó $A=\left(x_1+x_2\right)^2-3 x_1 x_2=(2 m)^2-3(-2 m-3)=$ $4 m^2+6 m+9$.

$A=(2 m)^2+2.2 m \cdot \frac{3}{2}+\frac{9}{4}+\frac{27}{4}=\left(2 m+\frac{3}{2}\right)^2+\frac{27}{4} \geq \frac{27}{4}$. Đẳng thức xảy ra khi $m=\frac{-3}{4}$.

Vậy giá trị nhỏ nhất của $A$ là $\frac{27}{4}$ khi $m=\frac{-3}{4}$.

Bài tập rèn luyện

Bài 1. Cho phương trình $x^2-\sqrt{2} x-\sqrt{3}=0$.
(a) Không giải phương trình, chứng minh phương trình có hai nghiệm $x_1, x_2$.
(b) Tính giá trị của $A=x_1^2+x_2^2-3 x_1 x_2 .(A=2+5 \sqrt{3})$
(c) Tính giá trị của biểu thức $B=\frac{1}{x_1^3-4 x_1 x_2+x_2^3}$
Bài 2. Cho phương trình $x^2-2 m x-1=0$.
(a) Chứng minh rằng phương trình luôn có hai nghiệm phân biệt với mọi $m$
(b) Gọi $x_1, x_2$ là nghiệm của phương trình. Tính $A=$ $x_1^2-3 x_1 x_2+x_2^2$ theo $m$. $\left(A=4 m^2+5\right)$
(c) Tìm $m$ để $A=9 .(m= \pm 1)$
Bài 3. Cho phương trình $x^2-2(m-3) x-2 m+5=0$.
(a) Chứng minh rằng phương trình luôn có nghiệm $x_1, x_2$.
(b) Tìm $m$ để $x_1^2+x_2^2-3 x_1 x_2+x_1+x_2=17$. $\left(m=\frac{3 \pm \sqrt{21}}{2}\right)$

Bài 4. Cho phương trình $x^2-3(m+1) x+9 m^2+2=0$. Tìm $m$ để phương trình có hai nghiệm phân biệt $x_1, x_2$ thỏa $x_1^2+x_2^2-3\left(x_1+x_2\right)+1=0$.
(Không có giá trị $m$ nào thỏa mãn)
Bài 5. Cho phương trình $x^2-3 x-4 m=0$
(a) Tìm $m$ để phương trình có hai nghiệm phân biệt. $\left(m>\frac{-9}{16}\right)$
(b) Tìm $m$ để phương trình có hai nghiệm phân biệt $x_1, x_2$ thỏa $x_1+x_2-x_1 x_2=13\left(m=\frac{5}{2}\right)$
(c) Tính giá trị biểu thức $A=x_1^2+x_2^2-4 x_1 x_2$ theo $\mathrm{m}$ và tìm $\mathrm{m}$ để $\mathrm{A}=14$. $\left(A=9+24 m, m=\frac{5}{24}\right)$
Bài 6. Cho phương trình $x^2-2(m-1) x-1=0$.
(a) Chứng minh rằng phương trình luôn có hai nghiệm phân biệt $x_1, x_2$.
(b) Tìm $\mathrm{m}$ để $x_1^2+x_2^2=5\left(m=\frac{2 \pm \sqrt{3}}{2}\right)$
(c) Tìm giá trị nhỏ nhất của biểu thức $x_1^2+x_2^2+x_1 x_2$ (GTNN là 1 khi và chỉ khi $m=1$ )

Bài 7. Cho phương trình $x^2-2(m+1) x+m=0$
(a) Chứng minh rằng phương trình luôn có hai nghiệm phân biệt $x_1, x_2$
(b) Tìm m để $x_1^2+x_2^2-3 x_1 x_2-x_1-x_2=7$
$$
\left(m=\frac{-5 \pm \sqrt{41}}{8}\right)
$$
(c) Tìm giá trị nhỏ nhất của biểu thức $B=x_1^2+x_2^2$ $\left(B_{\min }=\frac{7}{4}\right.$ khi và chỉ khi $\left.x=\frac{-3}{4}\right)$
Bài 8. Cho phương trình $x^2-2 m x-m-3=0$.
(a) Tìm $m$ dể phương trình có hai nghiệm $x_1, x_2$ thỏa
$$
\begin{aligned}
& \frac{1}{x_1}+\frac{1}{x_2}+\frac{1}{2}=0 \
& \left(m=\frac{-3}{5}\right)
\end{aligned}
$$

(b) Tìm $m$ để phương trình có nghiệm thỏa $x_1^3-x_2^3=$ $10\left(x_1-x_2\right)$ $\left(m=\frac{-1 \pm \sqrt{113}}{8}\right)$
Bài 9. Cho phương trình $(m-1) x^2-2 x+1=0$.
(a) Tìm $m$ để phương trình có hai nghiệm phân biệt. $(m \neq 1, m>2)$
(b) Tìm $m$ để phương trình có hai nghiệm $x_1, x_2$ thỏa $x_1^2+x_2^2+x_1 x_2=3\left(m=\frac{-1}{3}\right)$
Bài 10. Cho phương trình $x^2+2(m+2) x+2 m=0$.
(a) Tìm $m$ để phương trình có hai nghiệm $x_1, x_2$ thỏa $x_1^2 x_2+x_2^2 x_1+x_1+x_2=4$
(không có giá trị $m$ thỏa mãn)
(b) Tìm giá trị lớn nhất của biểu thức $A=x_1 x_2-x_1^2-$ $x_2^2\left(A_{\max }=\frac{-63}{4}\right.$ khi và chỉ khi $\left.m=\frac{-1}{4}\right)$

Hệ phương trình ba ẩn

Trong các bài trước mình đã làm quen với các hệ phương trình hai ẩn, phương pháp chủ yếu cũng là thế, cộng đại số, đặt ẩn phụ. Trong bài này chúng ta tiếp tục với các hệ phương trình nhiều ẩn hơn, chủ yếu là các hệ phương trình ba ẩn, trong các hệ phương trình này có hai dạng ta quan tâm và xuất hiện nhiều là hệ đối xứng và hệ hoán vị vòng quanh.

Hệ ba ẩn đối xứng

Hệ đối xứng ba biến là hệ có dạng

$\left\{\begin{array}{l}
f(x,y,z)=0 \\\\
g(x,y,z)=0 \\\\
h(x,y,z)=0
\end{array}\right.$

trong đó $f, g, h$ là các biểu thức đối xứng với $x, y, z$ tức là khi ta hoán vị $x, y, z$ thì $f, g, h$ vẫn không đổi.

Các biểu thức đối xứng 3 biến cơ bản nhất là $x+y+z, xy+yz+xz, xyz$.

Từ đó ta xét ví dụ sau

Ví dụ 1. Giải hệ phương trình $\left\{\begin{array}{l}
x+y+z=6 (1)\\\\
xy+yz+xz=11 (2)\\\\
xyz=6 (3)
\end{array}\right.$

Lời giải

Từ (1) ta có $y +z = 6-z$, từ (2), $ yz = 11-x(y+z) = 11 – x(6-x) = x^2-6x+11$.

Thế vào (3) ta có $x(x^2-6x+11) = 6$ $\Leftrightarrow x^3 -6x^2+ 11x – 6 = 0$

Giải ra được $x = 1, x = 2, x= 3$.

Với $x = 1$ ta có $y+z = 5, yz = 6$ giải ra được $y = 2, z= 3$ và $y=3, z=2$.

Các trường hợp khác tương tự, hệ phương trình có nghiệm $(1, 2, 3)$ và các hoán vị.

Do đó nếu hệ phương trình ba ẩn đối xứng, có một cách giải là ta tìm được giá trị của các biểu thức đối xứng cơ bản như bài trên.

Ví dụ 2. (PTNK Chuyên toán 2010) Giải hệ phương trình $\left\{\begin{array}{l}
x+y+z=3 \\\\
x y+y z+x z=-1 \\\\
x^3+y^3+z^3+6=3\left(x^2+y^2+z^2\right)
\end{array}\right.$

Lời giải

Ta chỉ cần tính được $xyz$ thì có thể đưa về ví dụ 1.

Từ (1) và (2) ta tính được $x^2+y^2+z^2 = (x+y+z)^2 – 2(xy+yz+xz) = 11$

Suy ra $x^3+y^3+z^3 = 27$

Mà $x^3+y^3+z^3 – 3xyz=(x+y+z)(x^2+y^2+z^2-xy-yz-xz) \Rightarrow xyz = -3$

do đó ta có $x+y+z = 3, xy+yz+xz = -1, xyz = -3$ tương tự ví dụ 1, ta giải được nghiệm là $(1,-1,3)$ và các hoán vị.

Ngoài cách trên ta có thể giải như sau

$x^3+y^3+z^3 = (x+y+z)^3 – 3(x+y)(y+z)(x+z)$, khi đó $(x+y)(y+z)(z+x) = 0$, tổng hai số bằng 0, ta suy ra số còn lại bằng 3, tiếp tục ta cũng có kết quả như trên.

Hệ hoán vị vòng quanh

Các hệ phương trình nhiều ẩn thường gặp là hệ hoán vị vòng quanh có dạng sau:

Phương pháp thường dùng là cộng đại số,phân tích thành tích, sử dụng đánh giá bất đẳng thức để chứng minh $x=y=z$.

Ta xét một số ví dụ sau:

Ví dụ 3. Giải hệ phương trình $\left\{\begin{array}{l}(x-y)^2=2 z-z^2(1) \\\\(y-z)^2=2 x-x^2(2)\\\\ (z-x)^2=2 y-y^2(3)\end{array}\right.$

Lời giải Lấy (1) trừ (2) ta có:

$(x-2 y+z)(x-z)=x^2-z^2-2(x-z)=(x-z)(x+z-2) \Leftrightarrow 2(x-z)(y-1)= 0$

$\Leftrightarrow x=z$ hoặc $y=1$
– $y=1$ ta có $(3) \Leftrightarrow(x-z)^2=1 \Leftrightarrow z=x+1, z=x-1$
+ $z=x+1$ giải được $ x=0, z=1$ và $x=1, z=2 $Khi đó ta có nghiệm $(0,1,1),(1,1,2)$
+ $z=x-1 $ giải ra được $x=1, z=0 $ và $ x=2, z=1 $Ta có nghiệm $(1,1,0)$ và $(2,1,1)$
Với $x=z$ từ (3) ta có $ y^2-2 y=0 \Leftrightarrow y=0, y=2$

Với $y=0$ ta có $\left\{\begin{array}{l}x^2=2 z-z^2 \\\\ z^2=2 x-x^2\end{array} \Leftrightarrow \left\{\begin{array}{l}2 z^2=2 z \\\\ x-z\end{array}\right.\right.$.

Giải được nghiệm $(0,0,0)$ và $(1,0,1)$.

+Với $y=2$, giải ra được nghiệm $(1,2,1)$ và $(2,2,2)$. Vậy hệ phương trình có 8 nghiệm.

Ví dụ 4. (PTNK Chuyên Toán 2103) Giải hệ phương trình $\left\{\begin{array}{l}
3 x^2+2 y+1=2 z(x+2) \\\\
3 y^2+2 z+1=2 x(y+2) \\\\
3 z^2+2 x+1=2 y(z+2)
\end{array}\right.$

Lời giải Cộng ba phương trình lại ta có:
$3\left(x^2+y^2+z^2\right)+2(x+y+z)+3=2(x y+y z+z x)+4(x+y+z) $

$ \Leftrightarrow 3\left(x^2+y^2+z^2\right)-2(x y+y z+x z)-2(x+y+z)+3=0 $
$\Leftrightarrow(x-y)^2+(y-z)^2+(z-x)^2+(x-1)^2+(y-1)^2+(z-1)^2=0 $
$\Leftrightarrow\left\{\begin{array}{l}
x=1 \\\\
y=1 \\\\
z=1
\end{array}\right.
$
Thử lại thấy $(1,1,1)$ là nghiệm của hệ.

Ví dụ 5. Giải hệ phương trình $\left\{\begin{array}{l}
2 x=y^2-z^2 \\\\
2 y=z^2-x^2 \\\\
2 z=x^2-y^2
\end{array}\right.$

Lời giải

Lấy (1) $+(2)$ ta có $(x+y)(x-y+2)=0 \Leftrightarrow x+y=0$ hoặc $x=2-y$.
Với $x+y=0$, từ (3) ta có $z=0$, từ (1) ta có $x=0$ hoặc $x=2$. Ta có nghiệm $(x, y, z)$ là $(0,0,0)$ và $(2,-2,0)$.
Với $x=y-2$, từ (3) ta có $2 z=(y-2)^2-y^2=4-4 y \Leftrightarrow z=2-2 y$. Thế vào (1) ta có: $2(y-2)=y^2-(2-2 y)^2 \Leftrightarrow y^2-2 y=0 \Leftrightarrow y=0, y=2$. Từ đó ta có nghiệm $(-2,0,2)$ và $(2,-2,0)$. Vậy hệ có 4 nghiệm.

Hệ nhiều ẩn không mẫu mực

Một số hệ không mẫu mực thì không có cách giải chung, do đó ta phải để đặc điểm của các hệ phương trình này để có cách giải phù hợp, chủ yếu cũng là giảm được ẩn, phân tích nhân tử, . ..

Ví dụ 6. Giải hệ phương trình sau: $\left\{\begin{array}{l}
(x-2 y)(x-4 z)=55 \\\\
(y-2 z)(y-4 x)=-39 \\\\
(z-2 x)(z-4 y)=-16
\end{array}\right.$

Lời giải

$\left\{\begin{array}{l}(x-2 y)(x-4 z)=55 \\\\ (y-2 z)(y-4 x)=-39 \\\\ (z-2 x)(z-4 y)=-16\end{array} \Leftrightarrow\left\{\begin{array}{l}x^2-2 x y-4 x z+8 y z=55(1) \\\\ y^2-2 y z-4 x y+8 x z=-39(2) \\\\ z^2-2 x z-4 y z+8 x y=-16(3)\end{array}\right.\right.$

Cộng (1),(2),(3) ta có $(x+y+z)^2=0 \Leftrightarrow x+y+z=0 \Leftrightarrow z=-x-y$
Thế vào (1),(2) ta có $\left\{\begin{array}{l}(x-2 y)(5 x+4 y)=55 \\\\ (3 y+2 x)(y-4 x)=-39\end{array}\right.$

$\Leftrightarrow\left\{\begin{array}{l}5 x^2-6 x y-8 y^2=55 \\\\ 3 y^2-10 x y-8 x^2=-39\end{array}\right.$
Nhận thấy $y=0$ không thỏa hpt:
Đặt $x=k y$, ta có hệ

$\left\{\begin{array}{l}
y^2\left(5 k^2-6 k-8\right)=55 \\\\
y^2\left(-8 k^2-10 k+3\right)=-39
\end{array}\right. $
$\Rightarrow-39\left(5 k^2-6 k-8\right)=55\left(-8 k^2-10 k+3\right) $
$\Leftrightarrow 245 k^2+784 k+147=0$
$ \Leftrightarrow\left[\begin{array}{l}
k=-3 \\\\
k=\frac{-1}{5}
\end{array}\right.
$
Với $k=-3$, ta có $y=1$, hoặc $y=-1$. Từ đó ta có nghiệm là $(-3,1,2),(3,-1,-2)$
Với $k=-\frac{1}{5}$ (vô nghiệm)

Chìa khóa trong lời giải này chính là đặc điểm của các hệ số tự do bên phải của các phương trình.

Qua một số ví dụ , hi vọng các em rút ra kinh nghiệm trong việc giải một số hệ phương trình nhiều ẩn, cùng rèn luyện các bài toán sau nhé.

Bài tập rèn luyện

Bài 1. Giải các hệ phương trình sau

1)$\begin{cases} x^2(y+z)^2=(3x^2+x+1)y^2z^2&\\\\y^2(z+x)^2=(4y^2+y+1)z^2x^2&\\\\z^2(x+y)^2=(5z^2+z+1)=x^2y^2 \end{cases}$ 2)$\left\{ \begin{array}{l}xy = x + 3y\\\\yz = 2\left( {y + z} \right)\\\\xz = 3\left( {3z + 2x} \right)\end{array} \right.$ 3) $\left\{ \begin{array}{l}
{\left( {x + y + z} \right)^3} = 12t\\\\
{\left( {y + z + t} \right)^3} = 12x\\\\
{\left( {z + t + z} \right)^3} = 12y\\\\
{\left( {t + x + y} \right)^3} = 12z
\end{array} \right.$

Bài 2. Giải hệ phương trình sau:

1)$\left\{\begin{array}{l}
x^{3}+x^{2}+x-2=y \\\\
y^{3}+y^{2}+y-2=z \\\\
z^{3}+z^{2}+z-2=x
\end{array}\right.$
2) $\left\{\begin{array}{l}
y^{3}-6 x^{2}+12 x-8=0 \\\\
z^{3}-6 y^{2}+12 y-8=0 \\\\
x^{3}-6 z^{2}+12 z-8=0
\end{array}\right.$
Bài 3. Giải hệ phương trình $\begin{cases}ab+c+d=3&\\\\bc+d+a=5&\\\\cd+a+b=2&\\\\da+b+c=6 \end{cases}$

Bài 4.

Cho $a \in \mathbb{R}$. Giải hệ phương trình $\begin{cases} x_1^2+ax_1+(\dfrac{a-1}{2})^2=x_2&\\\\
x_2^2+ax_2+(\dfrac{a-1}{2})^2=x_3&\
…&\\\\
x_n^2+ax_n+(\dfrac{a-1}{2})^2=x_1
\end{cases}$

Hệ phương trình chứa tham số

Hệ phương trình và các phương pháp giải của nó chúng ta đã nghiên cứu trong các bài giảng trước, bài viết này ta tiếp tục với các hệ phương trình nhưng chứa thêm tham số, việc giải các hệ phương trình chứa tham số căn bản cũng dựa trên các phương pháp đã biết, tuy vậy ta phải xét nhiều trường hợp hơn đòi hỏi suy luận tốt và sự cẩn thận nhất định của học sinh.

Ví dụ 1. Cho hệ phương trình: $\left\{\begin{array}{l}\sqrt{x-2}+\sqrt{y-1}=2 \\\\ x+y=m\end{array}\right.$
(a) Giải hệ với $m=7$
(b) Tìm $m$ sao cho hệ có nghiệm $(x, y)$

Lời giải
a) $\left\{\begin{array}{l}\sqrt{x-2}+\sqrt{y-1}=2 \\\\ x+y=m\end{array}\right.$
ĐKXĐ: $x \geq 2, y \geq 1$
Đặt $ a=\sqrt{x-2}, b = \sqrt{y-1}$ ta có $a, b \geq 1$ và $a+b = 2, a^2+b^2 = 4$.

Từ đó ta có $b = 2-a, a^2+(2-a)^2 = 4$, giải ra được $a= 2, b=0$ và $a=0, b=2$.

Với $a = 2,b=0$ ta có $x=6, y=1$

Với $a=0,b=2$ ta có $x=2, y = 5$.

Vậy hệ phương trình có hai nghiệm $(2 ; 5),(6 ; 1)$

b) Đặt $u=\sqrt{x-2}, v=\sqrt{y-1}(u, v \geq 0$
Hệ phương trình trở thành: $\left\{\begin{array}{l}u+v=2 \\\\ u^2+v^2=m-3\end{array}\right.$ $\Rightarrow 2 u^2-4 u+7-m=0 \quad(2)$
Để hệ (1) có nghiệm thì (2) phải có nghiệm không âm, nhỏ hơn hoặc bằng 2, khi và chỉ khi:
$$
\left\{\begin{array} { l }
{ \Delta ^ { \prime } \geq 0 } \\\\
{ S > 0 } \\\\
{ P \geq 0 } \\\\
{ ( x _ { 1 } – 2 ) ( x _ { 2 } – 2 ) > 0 } \\\\
{ S \leq 4 }
\end{array} \Leftrightarrow \left\{\begin{array}{l}
m \geq 5 \\\\
m \leq 7
\end{array}\right.\right.
$$
Vậy $5 \leq m \leq 7$ thì hệ đã cho có nghiệm $(x, y)$

Ví dụ 2. Giải và biện luận hệ phương trình sau: $\left\{\begin{array}{l}\frac{x y z}{x+y}=m \\\\ \frac{x y z}{y+z}=1 \ \frac{x y z}{z+x}=2\end{array}\right.$

Lời giải

Lời giải. Đặt $a=x y, b=y z, c=x z$ ta tính được: $\frac{1}{a}=\frac{3 m-2}{4 m}, \frac{1}{b}=\frac{m+2}{4 m}, \frac{1}{c}=\frac{2-m}{4 m}$.
Khi đó $\frac{1}{(x y z)^2}=\frac{1}{a b c}=\frac{(3 m-2)(m+2)(2-m)}{64 m^3}=P$.
Nếu $P \leq 0 \Leftrightarrow m \leq-2,0 \leq m \leq \frac{2}{3}$ hoặc $m \geq 2$ thì hệ vô nghiệm.
Ta có $P>0 \Leftrightarrow-2<m<0$ hoặc $\frac{2}{3}<m<2$.
Khi đó $(x y z)^2=\frac{64 m^3}{(3 m-2)(m+2)(2-m)}=\frac{1}{P}$. Suy ra $x y z= \pm \sqrt{\frac{1}{P}}$.

  • Nếu $x y z=\sqrt{\frac{1}{P}}$ thì $x=\frac{2-m}{4 m} \sqrt{\frac{1}{P}}$,
    $$
    y=\frac{m+2}{4 m} \sqrt{\frac{1}{P}}, z=\frac{3 m-2}{4 m} \sqrt{\frac{1}{P}} \text {. }
    $$
  • Nếu $x y z=-\sqrt{\frac{1}{P}}$ thì $x=\frac{m-2}{4 m} \sqrt{\frac{1}{P}}$,
    $$
    y=\frac{-m-2}{4 m} \sqrt{\frac{1}{P}}, z=\frac{2-3 m}{4 m} \sqrt{\frac{1}{P}} \text {. }
    $$

Ví dụ 3. Cho hệ phương trình $\left\{\begin{array}{l}(x-2 y)(x+m y)=m^2-2 m-3 \\\\ (y-2 x)(y+m x)=m^2-2 m-3\end{array}\right.$

a) Giải hệ phương trình khi $m=-3$

b) Tìm $m$ để hệ có ít nhất một nghiệm $\left(x_\circ, y_\circ \right)$ thỏa $x_\circ>0, y_\circ>0$.

Lời giải
a) Khi $m=-3$ ta có hệ:
$$
\left\{\begin{array} { l }
{ ( x – 2 y ) ( x – 3 y ) = 1 2 } \\\\
{ ( y – 2 x ) ( y – 3 x ) = 1 2 }
\end{array} \Leftrightarrow \left\{\begin{array}{l}
x^2-5 x y+6 y^2=12(1) \\\\
y^2-5 x y+6 x^2=12(2)
\end{array}\right.\right.
$$
Lấy (1) – (2) ta có $5\left(y^2-x^2\right)=0 \Leftrightarrow x=y, x=-y$.
Với $x=y$ thế vào (1) ta có $x^2=6 \Leftrightarrow x=\sqrt{6}, y=\sqrt{6}$ hoặc $x=-\sqrt{6}, y=$ $-\sqrt{6}$
Với $x=-y$ thế vào (1) ta có $x^2=1 \Leftrightarrow x=1, x=-1$. Với $x=1, y=-1$, với $x=-1, y=1$.
Vậy hệ phương trình có 4 nghiệm.
b) Hệ có thể viết lại $\left\{\begin{array}{l}x^2+(m-2) x y-2 m y^2=m^2-2 m-3(1) \\\\y^2+(m-2) x y-2 m x^2=m^2-2 m-3(2)\end{array}\right.$
Lấy (1) – (2) ta có $(2 m+1)\left(y^2-x^2\right)=0$.
Xét $m=\frac{-1}{2}$ ta có hệ trở thành: $x^2-\frac{5}{2} x y+y^2+\frac{7}{4}=0$, có nghiệm $\left(\frac{5+\sqrt{2}}{2}, 2\right)$ thỏa đề bài.
Xét $m \neq \frac{-1}{2}$ ta có $x=y$ hoặc $x=-y$. Trường hợp $x=-y$ không thỏa đề bài.
Trường hợp $x=y$, thế vào (1) ta có:
$$
-(m+1) x^2=m^2-2 m-3=(m+1)(m-3)
$$
Nếu $m=-1$ ta có $(x-2 y)(x-y)=0,(y-2 x)(y-x)=0$ có nghiệm thỏa đề bài, chỉ cần chọn $x=1, y=1$.
Nếu $m \neq-1$ ta có $x^2=3-m$ để có nghiệm $x_o=y_o>0$ thì $m<3$. Khi đó phương trình có nghiệm $x_0=\sqrt{3-m}, y_o=\sqrt{3-m}$ thỏa đề bài.
Kết luận $m=\frac{-1}{2}, m=-1$ và $m<3$.

Ví dụ 4. Cho hệ phương trình với $k$ là tham số:
$$\left\{\begin{array}{l}
\frac{x}{\sqrt{y z}}+\sqrt{\frac{x}{y}}+\sqrt{\frac{x}{z}}=k \\\\
\frac{y}{\sqrt{z x}}+\sqrt{\frac{y}{z}}+\sqrt{\frac{y}{x}}=k \\\\
\frac{z}{\sqrt{x y}}+\sqrt{\frac{z}{x}}+\sqrt{\frac{z}{y}}=k
\end{array}\right.
$$
(a) Giải hệ với $k=1$.
(b) Chứng minh hệ vô nghiệm với $k \geq 2$ và $k \neq 3$.

Lời giải

Điều kiện xác định là: $x, y, z$ cùng dương hoặc cùng âm.
Đặt $a=\sqrt{\frac{x}{y}}, b=\sqrt{\frac{y}{z}}, c=\sqrt{\frac{z}{x}}$ thì $a, b, c>0$ và $a b c=1$.
Ta có: $\frac{a}{c}=\frac{|x|}{\sqrt{y z}}, \frac{b}{a}=\frac{|y|}{\sqrt{z x}}, \frac{c}{b}=\frac{|z|}{\sqrt{x y}}$.
a) Khi $k=1$, nếu $x, y, z>0$ thì $\frac{a}{c}+a+\frac{1}{c}=\frac{b}{a}+b+\frac{1}{a}=\frac{c}{b}+c+\frac{1}{b}=1$.
Cộng lại suy ra $\left(a+\frac{1}{a}\right)+\left(b+\frac{1}{b}\right)+\left(c+\frac{1}{c}\right)+\left(\frac{c}{c}+\frac{b}{a}+\frac{c}{b}\right)=3$
Theo bất đẳng thức Cô-si thì rõ ràng $a+\frac{1}{a} \geq 2, b+\frac{1}{b} \geq 2, c+\frac{1}{c} \geq 2$ nên đẳng thức trên không thể xảy ra.
Xét trường hợp $x, y, z$ cùng âm thì
$$
-\frac{a}{c}+a+\frac{1}{c}=-\frac{b}{a}+b+\frac{1}{a}=-\frac{c}{a}+c+\frac{1}{b}=1
$$
Trừ vào các vế và phân tích, ta suy ra:
$$
\frac{(a-1)(b-1)}{a}=\frac{(b-1)(c-1)}{b}=\frac{(c-1)(a-1)}{c}=0
$$
Từ đây dễ dàng suy ra ít nhất 2 trong $a, b, c$ phải là 1 mà $a b c=1$ nên $a=b=c=1$. Vì thế nên thay vào ta có $x=y=z<0$. Và mọi bộ số như thế đều thỏa mãn hệ.

b) Với $k \geq 2$, giả sử hệ có nghiệm $(x, y, z)$. Nếu như $x, y, z<0$ thì ta có $\frac{(a-1)(b-1)}{a}=\frac{(b-1)(c-1)}{b}=\frac{(c-1)(a-1)}{c}=k-1>0$.
Từ đó suy ra $a-1, b-1, c-1$ dều cùng dấu, kéo theo $a, b, c>1$ hoặc $a, b, c<1$ Tuy nhiên $a b c=1$ nên điều này không thể xảy ra. Do đó, ta phải có $a, b, c>0$ nên đưa về
$$
\frac{a}{c}+a+\frac{1}{c}=\frac{b}{a}+b+\frac{1}{a}=\frac{c}{b}+c+\frac{1}{b}=k
$$
Trong các số $a, b, c$ giả sử $a=\max {a, b, c}$ thì $k=\frac{a}{c}+a+\frac{1}{c} \geq$ $\frac{a}{c}+2 \sqrt{\frac{a}{c}} \geq 1+2=3$ nên ta cần có $k \geq 3$. Vì $k \neq 3$ nên $k>3$.
Vì $a=\max {a, b, c} \geq 1$ nên ta có $2 b+1 \geq \frac{b}{a}+b+\frac{1}{a}=k>3$ kéo theo $b>1$. Tương tự từ $2 c+1>\frac{c}{b}+c+\frac{1}{b}=k>3$ nên $c>1$. Từ đây suy ra $a, b, c>1$ trong khi $a b c=1$, vô lý.
Vậy hệ luôn vô nghiệm với $k \geq 2$ và $k \neq 3$.

Bài tập rèn luyện

Bài 1. Cho hê phương trình $\left\{\begin{array}{l}x+y=m-2 \\\\x^2+y^2+2 x+2 y=-m^2+4\end{array}\right.$ (trong đó $m$ là tham số $x$ và y là ẩn)
a) Tìm $m$ để hệ phương trình trên có nghiệm.
b) Tìm giá trị lớn nhất, nhỏ nhất của biểu thúc $A=x y+2(x+y)+2011$.

Bài 2. Cho hệ phương trình $\left\{\begin{array}{c}x^2+y^2+x y=m^2-2 m+4 \\\\ x^2+y^2-3 x y=5 m^2-10 m+4\end{array} \quad\right.$ (m là tham số)
a) Giải hệ phương trình khi $m=-1$.
b) Chứng minh rằng hệ phương trình luôn có nghiệm với mọi giá trị của $m$. Tìm $m$ để phương trình có nghiệm $(x ; y)$ thỏa $y>x>0$ và $5 x^2-2 x y+y^2$ đạt giá trị nhỏ nhất.

Bài 3. Tìm $a$ để hệ phương trình
$\left\{\begin{array}{c}
& \frac{a x+y}{y+1}+\frac{a y+x}{x+1}=a \\\\
& a x^2+a y^2=(a-2) x y-x
\end{array} \quad\right.$
có nghiệm duy nhất.

CHUYÊN ĐỀ SỐ HỌC : PHÉP CHIA HẾT VÀ PHÉP CHIA CÓ DƯ

MỘT SỐ VÍ DỤ

 

Ví dụ 1

Cho $a$ là một số nguyên. Tìm UCLN $(2 a+3,3 a+4)$.

Lời giải

Gọi $d=(2 a+3,3 a+4)$, ta có $d \backslash 2 a+3$ và $d \backslash 3 a+4$.

Vì $3(2 a+3)-2(3 a+4)=1$ nên $d$ là ước của 1 hay $d=1$.

Ví dụ 2

Cho $\mathrm{a}, b$ là các số nguyên dương sao cho $a^2+b^2$ chia hết cho tích $a . b$. Hãy tính giá trị của biểu thức

$A=\frac{a^2+b^2}{a b} .$

(Thi học sinh giỏi Toán 9 – Thành phố Hà Nội, năm 2002).

Lời giải

Gọi $d=(a, b)$ thì $a=d . a_1$ và $b=d . b_1$ với $\left(a_1, b_1\right)=1$. Ta có :

$a^2+b^2=d^2\left(a_1^2+b_1^2\right) \text { và } a b=d^2 a_1 b_1 .$

  • Vì $a^2+b^2$ chia hết cho $a b$ nên $a_1^2+b_1^2$ chia hết cho $a_l b_1$. Suy ra $a_1^2+b_1^2$ chia hết cho $a_l$ và $b_l$. Suy ra $a_1^2$ chia hết cho $b_l$ và $b_1^2$ chia hết cho $a_l$.
  • Vì $\left(a_1, b_1\right)=1$ nên $\mathrm{a}_1$ chia hết cho $\mathrm{b}_1$ và $\mathrm{b}_1$ chia hết cho $\mathrm{a}_1$.

Suy ra $a_l=b_1=1$. Vậy,

$A=\frac{d^2\left(a_1^2+b_1^2\right)}{d^2 c_1 b_1}=\frac{2 d^2 a_1^2}{d^2 c_1^2}=2$

Ví dụ 3

Chứng minh rằng với mọi số nguyên dương $n$ ta đều có $n^3+5 n$ chia hết cho 6 .

(Thi vào lớp 10 chuyên, DHKHTN ĐHQGHN năm 1996).

Lời giải

Ta có $n^3+5 n=\left(n^3-n\right)+6 n$. Để chứng minh $n^3+5 n$ chia hết cho 6 ta chứng minh $n^3-n$ chia hết cho 6 .

Do $n^3-n=n(n-1)(n+1)$ là tích của ba số nguyên liên tiếp nên chia hết cho 2 và 3 .

Vì $(2,3)=1$ nên $n^3-n$ chia hết cho tích $2 \times 3=6$.

Ví dụ 4

Cho $a, b, c$ là các số nguyên. Chứng minh rằng $a^3+b^3+c^3$ chia hết cho 6 khi và chỉ khi $a+b+c$ chia hết cho 6 .

Lời giải

Xét $A=a^3+b^3+c^3-a-b-c=\left(a^3-a\right)+\left(b^3-b\right)+\left(c^3-c\right)$.

Theo ví dụ 3 thì $a^3-a \cdot b^3-b$ và $c^3-c$ đều chia hết cho 6 . Suy ra $A$ chia hết cho 6. Vậy. $a^3+b^3+c^3$ chia hết cho 6 khi và chỉ khi $a+b+c$ chia hết cho 6 .

Ví dụ 5

Chứng minh $S=n^2+3 n-38$ không chia hết cho 49 , với mọi số tự nhiên $n$.

Lời giải

Giả sử tồn tại $n$ sao cho $S=n^2+3 n-38$ chia hết cho $+9$. Vì

$n^2-4 n+4=n^2+3 n-38-7(n-6)$

nên $n^2-t n+4$ chia hết cho 7 hay $(n-2)^2$ chia hết cho 7 . Suy ra $n-2$ chia hết cho 7 hay $n=2+7 t$.

Thay vào $S$ ta được : $S=49\left(t^2+t\right)-28$. Suy ra $S$ không chia hết cho 49 , trái với điều giả sử.

Vậy $S$ không chia hết cho 49 với mọi số tự nhiên $n$.

Ví dụ 6

Chứng minh rằng với mọi số tự nhiên $n$ ta luôn có

$A=2005^n+60^n-1897^n-168^n \text { chia hết cho } 2004 \text {. }$

Lời giải

Ta có $2004-12 \times 167$. Vì $(12,167)=1$ nên để chứng minh $A$ chia hết cho 2004 ta chứng minh $A$ chia hết cho 12 và 167 .

Ta có: $A=\left(2005^n-1897^n\right)-\left(168^n-60^n\right)$.

Áp dụng tính chất $a^{\prime \prime}-b^n$ chia hết cho $a-b$ với mọi $n$ tự nhiên và $a-b \neq 0$. ta suy ra $2005^n-1897^n$ chia hết cho $2005-1897=108=12 \times 9$.

Suy ra $2005^n-1897^n$ chia hết cho 12 . Mặt khác, 168 và 60 đều chia hết cho 12 nên $168^n-60^n$ chia hết cho 12 . Vậy $A$ chia hết cho 12 .

Tương tự như trên, ta có

$A=\left(2005^n-168^n\right)-\left(1897^n-60^n\right) .$

Cũng lập luận tương tự như trên, ta có $2005^n-168^n$ chia hết cho $2005-168=1837$; $1897^n-60^n$ chia hết cho $1897-60=1837$ và $1837=11 \times 167$ nên $2005^n-168^n$ và $1897^n-60^n$ chia hết cho 167 . Suy ra $A$ chia hết cho 167 .

Vậy ta có điều phải chứng minh.

BÀI TẬP

1. Chứng minh $a+2 \mathrm{~b}$ chia hết cho 3 khi và chỉ khi $b+2 a$ chia hết cho 3 .

2. Giả sử $a-c$ là ước của $a b+c d$. Chứng minh rằng $a-c$ cũng là ước của $a d+b c$.

3. Cho $a, b \in \mathbb{N}$. Chứng minh $\frac{11 a+2 b}{19} \in \mathbb{Z}$ khi và chỉ khi $\frac{18 a+5 b}{19} \in \mathbb{Z}$.

4. Cho $n$ nguyên dương. Chứng minh rằng

$(n !+1,(n+1) !+1)=1 .$

5. Cho $a, b$ là các số nguyên. Chứng minh rằng

$(5 a+3 b, 13 a+8 b)=(a, b) \text {. }$

6. Cho các số nguyên $m, n, p, q$ thỏa mãn $|p \cdot m-q \cdot n|=1$. Chứng minh rằng với mọi cặp số nguyên $a, b$ ta đều có

$(m a+n b, p a+q b)=(a, b) .$

7. Giả sử $(a, n)=p$ và $(b, n)=q$. Chứng minh rằng $(a b, n)=(p q, n)$.

8. Cho $a \leq b \leq c$ và $b=a q_1+r_1, c=a \cdot q_2+r_2$. Chứng minh rằng

$(a, b, c)=\left(a, r_1, r_2\right) .$

9. Chứng minh rằng với mọi số tự nhiên $n$, các phân số sau là phân số tối giản

(a) $\frac{21 n+4}{14 n+3}$;

(b) $\frac{15 n^2+8 n+6}{30 n^2+21 n+13}$;

(c) $\frac{n^3+2 n}{n^4+3 n^2+1}$.

$(I M O-1959)$.

10. Xác định các giá trị của $n$ để các phân số sau đây là phân số tối giản

(a) $\frac{n+22}{n+3}$

(b) $\frac{3 n+2}{2 n+3}$

(c) $\frac{18 n+3}{21 n+7}$.

11. Xét phân số

$A=\frac{n^2+4}{n+5} .$

Hỏi có bao nhiêu số tự nhiên $n$ trong khoảng từ 1 đến 2005 sao cho phân số $A$ chưa tối giản?

12. Chứng minh rằng với mọi bộ ba số lẻ $a, b, c$ ta đều có

$\left(\frac{a+b}{2}, \frac{b+c}{2}, \frac{c+a}{2}\right)=(a, b, c) .$

13. Cho $\mathrm{a}, \mathrm{b}, c$ là các số nguyên dương. Chứng minh

a) $(a, b, c)=\frac{(a, b, c) a b c}{(a, b)(b, c)(c, a)}$;

b) $[a, b, c]=\frac{(a, b, c)[a, b][b, c][c, a]}{a b c \ldots}$.

14. Cho $a_1, a_2, \ldots, a_n$ là các số nguyên dương và $n>1$. Đặt

$A=a_1 a_2 \ldots a_n, A_i=\frac{A}{a_i}(i=\overline{1, n}) .$

Chứng minh các đẳng thức sau :

a) $\left(a_1, a_2, \ldots, a_n\right)\left[A_1, A_2, \ldots, A_n\right]=A$;

b) $\quad\left[a_1, a_2, \ldots, a_n \mid\left(A_1, A_2, \ldots, A_n\right): A\right.$.

15. Cho $m, n$ là hai số tự nhiên nguyên tố cùng nhau. Hãy tìm ước số chung lớn nhất của hai số $A=m+n$ và $B=m^2+n^2$.

(Thi học sinh gioi Toán toàn quốc lớp 9 năm 1979).

16. Xác định ước số chung lớn nhất của hai số sau :

a) $(7 a+1,8 a+3)$

b) $(11 a+2,18 a+5)$

trong đó $a$ là một số nguyên cho trước.

17. Cho $n$ là một số nguyên dương. Hãy tính bội số chung nhỏ nhất của các số

$n, n+1, n+2 \text {. }$

18. Chứng minh rằng với mọi số nguyên dương $n$ ta có

$[1,2, \ldots, 2 n]=[n+1, n+2, \ldots, n+n]$

19. Cho số nguyên $a$ không chia hết cho 2 và 3 . Chứng minh rằng :

$A: 4 a^2+3 a+5 \text { chia hết cho } 6 \text {. }$

20. Chứng minh rằng $\frac{a}{3}+\frac{a^2}{2}+\frac{a^3}{6} \in \mathbb{Z}, \forall u \in \mathbb{Z}$.

21. Chứng minh rằng $\mathrm{A}(\mathrm{n})=\mathrm{n}^4+6 \mathrm{n}^3+11 \mathrm{n}^2+6 n$ chia hết cho 24 .

(Thi học sinh giỏi Toán toàn quốc – lớp 9 năm 1975)

22. Chứng minh rằng $n^5-n$ chia hết cho 30 , với mọi $n$.

23. Chứng minh rằng $m^3+3 m^2-m-3$ chia hết cho 48 , với mọi $m$ lẻ.

24. Chứng minh rằng $n^{12}-n^8-n^4+1$ chia hết cho 512 , với mọi $n$ lẻ.

25. Chứng minh rằng $A(n)=n^4 \cdots 14 n^3+71 n^2-154 n+120$ chia hết cho 24 , với mọi số tự nhiên $n$.

26. Chứng minh rằng $n^4-4 n^3-4 n^2+16 n$ chia hết cho 384 , với mọi số tự nhiên $n$ chẵn.

(Thi học sinh giỏi toàn quốc – lớp 9 năm 1970)

27. Tìm tất cả các số nguyên dương $n$ sao cho $n^2+9 n-2$ chia hết cho 11 .

28. Tìm tất cả các số nguyên $x$ sao cho : $\left(x^3-8 x^2+2 x\right)$ chia hết cho $x^2+1$.

(Thi vô địch Bun-ga-ri năm 1977)

29. Cho $f(x)=a x^2+b x+c$ thoả mãn : $f(x) \in \mathbb{Z}, \forall x \in \mathbb{Z}$. Hỏi $a, b, c$ có nhất thiết phải là các số nguyên hay không? Tại sao?

(Thi vào lớp 10 chuyên, $Đ H K H T N$ – ĐHQGHN năm 2001)

30. Chứng minh $n^2+n+2$ không chia hết cho 15 , với mọi $n$ thuộc $\mathbb{Z}$.

31. Chứng minh $n^2+3 n+5$ không chia hết cho 121 , với mọi $n$ thuộc $\mathbb{N}$.

32. Chứng minh $9 n^3+9 n^2+3 n-16$ không chia hết cho 343 , với mọi $n$ thuộc $\mathbb{N}$.

33. Chứng minh $4 n^3-6 n^2+3 n+37$ không chia hết cho 125 , với mọi $n$ thuộc $\mathbb{N}$.

34. Cho $a$ và $b$ thuộc $\mathbb{N}$. Chứng minh rằng $5 a^2+15 a b-b^2$ chia hết cho $49 \mathrm{khi}$ và chỉ khi $3 a+b$ chia hết cho 7 .

35. Cho $a, b \in \mathbb{N}$. Chứng minh rằng $2 a+b$ chia hết cho 7 khi và chỉ khi $3 a^2+10 a b-8 b^2$ chia hết cho 49 .

36. Cho $n \in \mathbb{N}$. Chứng minh rằng số $A=5^n\left(5^n+1\right)-6^n\left(3^n+2^n\right)$ chia hết cho 91 .

(Thi vào lớp 10 chuyên, ĐHSPHN năm 1998).

37. Cho $n \in \mathbb{N}$. Chứng minh $6^{2 n}+19^n-2^{n+1}$ chia hết cho 17 .

38. Chứng minh $2^{8 n} \cdot 5^{6 n}-1980^n-441^n+1$ chia hết cho 1979 , với mọi $n$ thuộc $\mathbb{N}$.

39. Chứng minh $118^n-101^n-16^n-1$ chia hết cho 234 , với mọi $n$ lẻ.

40. Chứng minh $11^{n+2}+12^{2 n+1}$ chia hết cho 133 , với mọi $n$ thuộc $\mathbb{N}$.

41. Chứng minh $5^{2 n-1} \cdot 2^{n+1}+3^{n+1} \cdot 2^{2 n-1}$ chia hết cho 38 , với mọi $n$ thuộc $\mathbb{N}^*$.

42. Chứng minh $5^{n+2}+26.5^n+8^{2 n+1}$ chia hết cho 59. với mọi $n$ thuộc $\mathbb{N}$.

43. Tìm số tự nhiên $n$ lớn nhất sao cho $: 29^n$ là ước của 2003 !.

44. Tìm số tự nhiên $k$ lớn nhất sao cho : $(1994 \text { ! })^{1995} \quad \vdots 1995^k$.

(Thi học sinh giỏi Toán toàn quốc – lớp 9. năm 199t).

45. Cho $n$ thuộc $\mathbb{N}$ và $n>3$. Chứng minh rằng nếu $2^n=10 a+b(0<b<10)$ thì tích $a \cdot b$ chia hết cho 6 .

(Thi học sinh giỏi Toán toàn quốc lớp 9 năm 1983).

46. Cho $n$ thuộc $\mathbb{N}, n \geq 1$. Chứng minh $T_n=1^5+2^5+\ldots+n^5$ chia hết cho tổng của $n$ số tự nhiên đầu tiên $S_n=1+2+\ldots+n$.

(Thi vào lớp 10 chuyên $Đ H S P H N$ năm 2001).

47. Tìm $n$ nguyên dương sao cho : $(n-1)$ ! chia hết cho $n$.

(Thi vô địch Hungari năm 1951).

48. Xác định $n$ nguyên dương $(\mathrm{n} \geq 3$ ) sao cho số $A=1.2 .3 \ldots \mathrm{n}$ (tích của $n$ số nguyên dương đầu tiên) chia hết cho $B=1+2+\ldots+n$.

(Thi vào lớp 10 chuyên ĐHKHTN – ĐHQGHN năm 1994).

49. Cho $a$ và $m$ là các số nguyên dương và $a>1$. Chứng minh rằng

$\left(\frac{a^m-1}{a-1}, a-1\right)=(m, a-1) .$

50. Cho $a, m, n$ là các số nguyên dương và $a \neq 1$. Chứng minh rằng $a^n-1 \backslash a^m-1$ khi và chỉ khi $n \backslash m$.

51. Cho $a, m, n$ là các số nguyên dương và $a>1$. Chứng minh rằng

$\left(a^m-1, a^n-1\right)=a^{(m, n)}-1 .$

52. Cho $a, b$ là hai số nguyên dương không nhỏ hơn 2 và nguyên tố cùng nhau. Chứng minh rằng nếu $m, n$ là hai số nguyên dương thỏa mãn $a^n+b^n \backslash a^m+b^m$ thì ta cũng có $n . \mid m$.

53. Cho $a, b, n$ là các số nguyên dương. Biết rằng với mọi số tự nhiên $k \neq b$ ta đều có $k^n-a$ chia hết cho $k-b$. Chứng minh $a=b^n$.

54. Chứng minh rằng tồn tại vô hạn số tự nhiên $n$ sao cho : $4 n^2+1$ chia hết cho cả 5 và 13 .

55. Giả sử $1-\frac{1}{2}+\frac{1}{3}-\ldots+\frac{1}{1319}=\frac{p}{q}$, trong đó $p, q$ là các số nguyên. Chứng minh rằng $p$ chia hết cho 1979.

56. Cho $a_1, a_2, \ldots, a_n \in{1,-1}, n \in \mathbb{N}^*$ và thoả mãn :

$a_1 a_2+a_2 a_3+\ldots+a_n a_1=0 \text {. }$

Chứng minh $n$ chia hết cho 4 .

57. Chứng minh rằng tổng bình phương của $p$ số nguyên liên tiếp ( $p$ là số nguyên tố, $p>3$ ) chia hết cho $p$.

58. Cho số nguyên $a$ không nhỏ hơn 2 . Hỏi có tồn tại hay không số tự nhiên $A$ sao cho

$a^{2001}<A<a^{2002}$

và $A$ có ít nhất 600 chữ số 0 ở tận cùng?

59. Có tồn tại hay không 4004 số nguyên dương sao cho tổng của 2003 số bất kì đều không chia hết cho 2003 .

(Balkan 2003).

60. Tìm một cặp số nguyên dương $(a, b)$ thoả mãn đồng thời các điều kiện sau :

a) $a b(a+b)$ không chia hết cho 7 .

b) $(a+b)^7-a^7-b^7$ chia hết cho $7^7$.

(IMO-198t).

61. Giả sử $a, b$ là hai số nguyên dương khác nhau. Chứng minh rằng tồn tại vô số số tự nhiên $n$ sao cho $a+n$ và $b+n$ là hai số nguyên tố cùng nhau.

 

LỜI GIẢI – HƯỚNG DẪN – ĐÁP SỐ

1. Suy ra từ đẳng thức : $(a+2 b)+(b+2 a)=3(a+b)$.

2. Suy ra từ đẳng thức : $(a b+c d)-(a d+b c)=(a-c)(b-d)$.

3. Suy ra từ đẳng thức : $5 \cdot \frac{11 a+2 b}{19}-2 \cdot \frac{18 a+5 b}{19}=a$.

4. Giả sử $d=(n !+1,(n+1) !+1)$.

Ta có $d \backslash n !+1$ và $d \backslash(n+1) !+1$ nên $d \backslash(n+1) !+1-n !-1=n ! . n\quad\quad(1)$.

Vì $d \backslash n !+1$ nên $(d, n)=(d, n !)=1$. Từ (1) suy ra $d=1$.

5. Giả sử $d=(a, b)$ và $d^{\prime}=(5 a+3 b, 13 a+8 b)$.

Vì $d \backslash a$ và $d \backslash b$ nên $d \backslash 5 a+3 b$ và $d \backslash 13 a+8 b$. Suy ra $d \backslash d\quad(1)$.

Vì $d^{\prime} \backslash 5 a+3 b$ và $d^{\prime} \backslash 13 a+8 b$ nên

$d^{\prime} \backslash 8(5 a+3 b)-3(13 a+8 b)=a$

và $\quad d^{\prime} \backslash 5(13 a+8 b)-13(5 a+3 b)=b$.

Suy ra $d^{\prime} \backslash d\quad\quad(2)$.

Từ (1) và (2) ta suy ra $d^{\prime}=d$.

6. Giải tương tự bài $1.5$.

7. Ta có $(a, n)=p$ nên $a=p \cdot a_1, n=p n_1$ với $\left(a_1, n_1\right)=1$. Suy ra

$(a b, n)=\left(p a_1 b, p n_1\right)=p \cdot\left(a_1 b, n_1\right)=p\left(b, n_1\right)=(p b, n)$

$\text { Vì }(b, n)=q \text { nên } b=q \cdot b_1 \text { và } n=q \cdot n_2 \text { với }\left(b_1, n_2\right)=1 . \text { Suy ra }$

$(p b, n)=\left(p \cdot q \cdot b_1, q \cdot n_2\right)=q\left(p b_1, n_2\right)=q\left(p, n_2\right)=(p q, n)$

8. Giải tương tự bài $1.5$.

9. a) Giả sử $(21 n+4,14 n+3)=d(d \geq 1)$.

Ta có $d \backslash 21 n+4$ và $d \backslash 14 n+3$ nên $d \backslash 3(14 n+3)-2(21 n+4)=1$.

Vậy $d=1$.

Các bạn tự giải các câu b) và c).

10. a) Ta có $\frac{n+22}{n+3}=1+\frac{19}{n+3}$. Phân số đã cho tói gian khi và chi khi $(n+3,19)=1$ hay $n \neq 19 m-3$.

b) Vì $(2 n+3,2)=1$ nên phân số đã cho tối gian khi và chi khi phân số sau tối gịản

$B=\frac{2(3 n+2)}{2 n+3}=3-\frac{5}{2 n+3} .$

Phân số $B$ tối giann khi và chi khi $(2 n+3,5)=1$.

Ta có $(2 n+3,5) \neq 1$ khi và chi khi $5 \backslash 2 n+3$ hay $2 n+3=5 a$.

Xét $2 n+3=5 a$, ta có $n=2 a+\frac{a-3}{2}$.

Vì $n$ và $a$ là các số nguyên nên $a-3=2 m$, từ đó có $n=5 m+6$.

Vậy phân số đã cho tối giản khi và chi khi $n \neq 5 m+6$.

c) Đáp số: $n \neq 7 m+1$.

11. Giả sử $A$ là phân số chưa tối giản. Đặt $d=\left(n^2+4, n+5\right)$ suy ra $d>1$. Ta có

$d \backslash(n+5)^2-\left(n^2+4\right)=10 n+21=10(n+5)-29$

nên $d \backslash 29$ suy ra $d=29$.

Ngược lại, nếu $n+5$ chia hết cho 29 thì có thể đặt

$n+5=29 . m\left(m \in \mathbb{N}^*\right)$

12. Giải tương tự bài 5 .

13. Giải tương tự bài 5 .

14. Giải tương tự bài 5 .

15. Giả sử $d=(A, B)(d \geq 1)$. Ta có $d \backslash A^2-B$ suy ra $d \backslash 2 m n\quad(1)$.

Vì $d \backslash A$ nên $d \backslash 2 n \cdot A$ hay $d \backslash 2 m n+2 n^2$. Suy ra $d \backslash 2 n^2\quad(2)$.

Tương tự ta cũng có $d \backslash 2 m^2\quad(3)$

Vì $(m, n)=1$ nên $m, n$ không cùng chẵn. Xét các trường hợp:

  • Nếu $m, n \cdot$ khác tính chã̃n lẻ thì $d$ lẻ. Từ (2) và (3) ta suy ra $d \backslash m^2$ và $d \backslash$ $n^2$. Vì $(m, n)=1$ nên $d=1$.

  • Nếu $m, n$ cùng lẻ thì $d$ chã̃n. Đặt $d=2 d$, từ (2) và (3) ta suy ra $d \backslash m^2$ và $d^n \backslash n^2$. Vì $(m, n)=1$ nên $d^n=1$. Suy ra $d=2$.

16. a) Đặt $d=(7 a+1,8 a+3)$.

Ta có $d \backslash 7(8 a+3)-8(7 a+1)=13$ nên $d=1$ hoặc $d=13$.

Để $d=13$ thì điều kiện cần và đủ là $13 \backslash 7 a+1$.

Xét phương trình: $7 a+1=13 x$.

Ta có $a=2 x-\frac{x+1}{7}$ là một số nguyên nên $7 \backslash x+1$.

Đặt $x+1=7 m$ ta được $a=13 m-2, m \in \mathbb{Z}$.

Vậy, khi $a=13 m-2, m \in \mathbb{Z}$ thì $(7 a+1,8 a+3)=13$,

$a \neq 13 m-2, m \in \mathbb{Z}$ thì $(7 a+1,8 a+3)=1$

b) Giải tương tự câu a).

Đáp Số:

  • Nếu $a=19 m-14, m \in \mathbb{Z}$ thì $(11 a+2,18 a+5)=19$

  • Nếu $a \neq 19 m-14, m \in \mathbb{Z}$ thì $(11 a+2,18 a+5)=1$.

18. Giả sử $m=[1,2, \ldots, 2 n]$ và $m^{\prime}=[n+1, \ldots, n+n](n \geq 2)$.

Để chứng minh $m=m^{\prime}$ ta chứng minh $m \backslash m^{\prime}$ và $m^{\prime} \backslash m$.

Vì $n+1, n+2, \ldots, n+n$ là ước của $m$ nên $m^{\prime} \backslash m$.

Ngược lại, xét số $a \in{1,2, \ldots, n}$ tùy ý.

Trong $a$ số nguyên liên tiếp $n+1, \ldots, n+a$ luôn có một số chia hết cho $a$ nên $a \backslash m^{\prime}$. Suy ra các số $1,2, \ldots, 2 n$ đều là ước của $m^{\prime}$ hay $m \backslash m^{\prime}$.

Vậy $m=m^{\prime}$.

19. Vì $a$ không chia hết cho 2 và 3 nên $a$ có dạng: $a=6 m \pm 1(m \in \mathbb{Z})$

  • Với $a=6 m+1$ ta có $A=4(6 m+1)^2+3(6 m+1)+5$

$=6\left(24 m^2+11 m+2\right) \vdots 6 \text {. }$

  • Với $a=6 m-1$ ta có $A=4(6 m-1)^2+3(6 m-1)+5$

$=6\left(24 m^2-5 m+1\right) \vdots 6 \text {. }$

Vậy $A$ chia hết cho 6 , với mọi $a$ không chia hết cho 2 và 3 .

20. Ta có $\frac{a}{3}+\frac{a^2}{2}+\frac{a^3}{6}=\frac{a(a+1)(a+2)}{6}$.

Vì $a(a+1)(a+2)$ là tích của ba số nguyên liên tiếp nên chia hết cho 6 từ đó suy ra đpcm.

21. Ta có $A(n)=n(n+1)(n+2)(n+3)$.

Vì tích của ba số nguyên liên tiếp chia hết cho 3 nên $A(n)$ chia hết cho 3 . Trong bốn số nguyên liên tiếp luôn có hai số chẵn liên tiếp, một trong hai số đó chia hết cho 4 nên $A(n)$ chia hết cho 8 .

Vì $(3,8)=1$ nên $A(n)$ chia hết cho $3 \times 8=24$.

22. Ta có $30=6 \times 5$. Vì $(6,5)=1$ nên để chứng minh $n^5-n$ chia hết cho 30 ta chứng minh $n^5-n$ chia hết cho 6 và 5 .

Ta có $n^5-n=(n-1) n(n+1)\left(n^2+1\right)$. Vì $(n-1) n(n+1)$ là tích ba số nguyên liên tiếp nên chia hết cho 2 và 3 .

Suy ra $n^5-n$ chia hết cho $2 \times 3=6$.

Mặt khác ta lại có

$n^5-n=(n-1) n(n+1)(n^2-4+5)$

$=(n-2)(n-1) n(n+1)(n+2)+5(n-1) n(n+1) .$

Vì $(n-2)(n-1) n(n+1)(n+2)$ là tích cua năm sổ nguyên liên tiếp nên chia hết cho 5 .

Suy ra $n^5-n$ chia hết cho 5 .

Vậy $n^5-n$ chia hết cho 30 .

23. Đặt $A=m^3+3 m^2-m-3$.

Ta có $A=(m+3)\left(m^2-1\right)=(m+3)(m+1)(m-1)$.

Vì $m$ lẻ nên $m=2 n+1(n \in \mathbb{Z})$, từ đó suy ra $A=8 .(n+2)(n+1) n \Rightarrow$ đpcm.

24. Đặt $A=n^{12}-n^8-n^4+1$. Ta có

$A=\left(n^4-1\right)\left(n^8-1\right)=\left[\left(n^2-1\right)\left(n^2+1\right)\right]^2\left(n^4+1\right) .$

Vì $n$ lẻ nên $n=2 m+1$, suy ra $A=64 \cdot[m(m+1)]^2\left(2 m^2+2 m+1\right)^2\left(n^4+1\right)$.

25. Ta có $24=3 \times 8$. Để chứng minh $A(n)$ chia hết cho 24 ta chứng minh $A(n)$ chia hết cho 3 và 8 .

Ta có $A(n)=(n-2)(n-3)(n-4)(n-5)$ (bạn đọc tự phân tích).

Vì $A(n)$ là tích của bốn số nguyên liên tiếp nên $A(n)$ chia hết cho 3 .

Trong bốn số nguyên liên tiếp $n-2, n-3, n-4, n-5$ luôn có hai số chã̃n liên tiếp. Một trong hai số đó chia hết cho 4 , số còn lại chia hết cho 2 nên $A(n)$ chia hết cho 8 . Vì $(3,8)=1$ nên $A(n)$ chia hết cho $3 \times 8=24$.

26. Đặt $A=n^4-4 n^3-4 n^2+16 n$. Ta có $A=n(n-4)\left(n^2-4\right)$.

Vì $n$ chẵn nên $n=2 m(m \in \mathbb{Z})$. Từ đó suy ra $A=16 .(m-2)(m-1) m(m+1)$.

Vì $(m-2)(m-1) m(m+1)$ là tích của 4 số nguyên liên tiếp nên chia hết cho 8 và 3 .

Từ đó có đpcm.

27. Đáp số: $n=11 m+6$ hoặc $n=11 m+7(m \in \mathbb{N})$.

Hướng dẫn :

$\text { Ta có } n^2+9 n-2 \vdots 11 \Leftrightarrow n^2-2 n-2 \vdots 11 \Leftrightarrow 4\left(n^2-2 n-2\right) \vdots 11$

$\Leftrightarrow 4 n^2-8 n+3 \vdots 11 \Leftrightarrow(2 n-1)(2 n-3) \vdots 11 .$

28. Đáp số: $x \in{-8,0,2}$.

Giả sử $\left(x^3-8 x^2+2 x\right) \vdots\left(x^2+1\right)$ suy ra

$x\left(x^2+1\right)-8\left(x^2+1\right)+x+8 \vdots\left(x^2+1\right) \text {. }$

hay $x+8 \vdots\left(x^2+1\right)\quad\quad( * )$

  • Nếu $x+8=0$ thì $x=-8$, thỏa mãn điều kiện đề bài.

  • Nếu $x \neq-8$ thì tù $\left(^*\right)$ ta phải có $|x+8| \geq x^2+1\quad\quad(1)$.

Bất phương trình (1) cho ta $x \in{-2,-1,0,1,2,3}$

Thử trực tiếp ta được $x=0$ và $x=2$ thỏa mãn.

Cách 2

$\text { Ta có } x+8 \vdots\left(x^2+1\right) \Rightarrow x^2+8 x \vdots\left(x^2+1\right) \Rightarrow 8 x-1 \vdots\left(x^2+1\right)$

$\Rightarrow  8(x+8)-(8 x-1) \vdots x^2+1 \Rightarrow 65 \vdots\left(x^2+1\right)$

$\Rightarrow x^2+1 \text { là ước dương của } 65$

$\Rightarrow x^2+1 \in{1,5,13,65} .$

29. Cho $x=0$ suy ra $f(0)=c \in \mathbb{Z}$. Các số $a, b$ không nhất thiết phải là các số nguyên.

Ví dụ, chọn $a=b=\frac{1}{2}$, ta có

$f(x)=\frac{x(x+1)}{2}+c \in \mathbb{Z}, \forall x \in \mathbb{Z} \text {. }$

30. Giả sử $n^2+n+2 \vdots 15$ ta có $n^2+n+2 \vdots 3\quad\quad(1)$.

Từ (1) suy ra $n$ không chia hết cho 3 .

Vậy $n$ có dạng $3 k+1$ hoặc $3 k-1(k \in \mathbb{Z})$, ta có

$n^2-1=(n-1)(n+1) \vdots 3$

$\Rightarrow n^2+n+2=\left(n^2-1\right)+n+3 \text { không chia hết cho } 3 \text {, mâu thuẫn với (1). }$

31. Giả sử $n^2+3 n+5 \vdots 121$ suy ra $n^2+3 n+5 \vdots 11$ hay $4 n^2+12 n+20 \vdots 11$ Vậy

$4 n^2+12 n+9 \vdots 11 \Rightarrow(2 n+3)^2 \vdots 11 \Rightarrow 2 n+3 \vdots 11$

Nhưng khi đó

$4\left(n^2+3 n+5\right)=(2 n+3)^2+11$ không chia hết cho 121 , mâu thuẫn với điều giả sử trên, từ đó suy ra đpcm.

32. Giải tương tự bài 31

33. Giải tương tự bài 31

34. $\Rightarrow \text { ) Giả sử } 5 a^2+15 a b-b^2 \vdots 49 \Rightarrow 5 a^2+15 a b-b^2 \vdots 7$

$\Rightarrow 9 a^2+6 a b+b^2 \vdots 7 \Rightarrow(3 a+b)^2 \vdots 7 \Rightarrow 3 a+b \vdots 7 \text {. }$

$\Leftrightarrow) \text { Giả sử } 3 a+b \vdots 7 \Rightarrow 3 a+b=7 c(c \in \mathbb{Z}) \Rightarrow b=7 c-3 a$

$\Rightarrow 5 a^2+15 a b-b^2=5 a^2+15 a(7 c-3 a)-(7 c-3 a)^2$

$=49\left(c^2+3 a c-a^2\right) \vdots 49 .$

35. Giải tương tự bài 34.

36. Ta có $91=7 \times 13$. Vì $(7,13)=1$ nên để chứng $\operatorname{minh} A \vdots 91$ ta chi cần chứng $\operatorname{minh} A \vdots 7$ và $A \vdots 13$.

  • Chứng $\operatorname{minh} A \vdots$ 7: Ta viết $A$ dưới dạng: $A=\left(25^n-18^n\right)-\left(12^n-5^n\right)$. Vì $\left(25^n-18^n\right) \vdots 25-18=7$ và $\left(12^n-5^n\right) \vdots 12-5=7$ nên $A \vdots 7$.

  • Chứng $\operatorname{minh} A \vdots$ 13: Ta viết $A$ dưới dạng: $A=\left(25^n-12^n\right)-\left(18^n-5^n\right)$ Vì $\left(25^n-12^n\right) \vdots 25-12=13$ và $\left(18^n-5^n\right) \vdots 18-5=13$ nên $A \vdots 13$. Vậy $A \vdots 91, \forall n \in \mathbb{N}$.

37. Đặt $A(n)=6^{2 n}+19^n-2^{n+1}$.

Ta có $A=36^n+19^n-2 \cdot 2^n=\left(36^n-2^n\right)+\left(19^n-2^n\right)$.

Vì $36^n-2^n \vdots 34(=36-2)$ nên $36^n-2^n \quad \vdots 17$ và $19^n-2^n \vdots 17(=19-2)$ nên $A(n) \vdots 17$.

38. Đặt $A(n)=2^{8 n} \cdot 5^{6 n}-1980^n-441^n+1$.

Ta có $A=\left(4000000^n-441^n\right)-(1980-1)^n$.

Vì $\left(4000000^n-441^n\right) \vdots 3999599(=4000000-441=2021)$ nên

$\left(4000000^n-441^n\right) \vdots 1979$

và $1980^n-1 \vdots 1979(=1980-1)$ nên $A(n) \vdots 1979$.

39. Giải tương tự bài $36$.

40. Giải tương tự bài $37$.

41. Giải tương tự bài 36.

42. Giải tương tự bài 37.

43. Các số chia hết cho 29 trong khoảng từ 1 đến 2003 là:

$29 \times 1,29 \times 2,29 \times 3, \ldots, 29 \times 69 \text {. }$

Suy ra $2003 !=29^{69} \cdot 69 ! . A$, trong đó $(A, 29)=1$.

Các số chia hết cho 29 trong khoảng từ 1 đến 69 là: $29 \times 1,29 \times 2$.

Suy ra: $69 !=29^2 \cdot 2 ! . B$, trong đó $(B, 29)=1$.

Vậy $2003 !=29^{71} \cdot 2 \cdot A \cdot B$, trong đó $(A \cdot B, 29)=1$.

Từ đó suy ra $n$ cần tìm là 71 .

44. Đáp số: $k=217455$.

$1995=3 \times 5 \times 7 \times 19 .$

Ta :cần tìm số mũ lớn nhất của 19 trong phân tích tiêu chuẩn của số $(1994 !)^{1995}$. Xem lại bài 43.

45. Ta có $2^n=10 a+b$ nên $b \vdots 2$ hay $a b \vdots 2$. Ta chứng minh $a b \vdots 3$ :

Từ đẳng thức $2^n=10 a+b$ suy ra $2^n$ có chữ số tận cùng là $b$.

Đặt $n=4 k+r(k, r \in \mathbb{N}, 0 \leq r \leq 3)$ ta có $2^n=16^k \cdot 2^r$.

Nếu $r=0$ thì $2^n=16^k$ có chữ số tận cùng là 6 suy ra $b=6$ hay $a b \vdots 6$.

Nếu $1 \leq r \leq 3$ thì $2^n-2^r=2^r\left(16^k-1\right) \vdots 10$ suy ra $2^n$ có tận cùng là $2^r$.

Vậy ta có $b=2^r$, từ đó suy ra

$10 a=2^n-2^r=2^r\left(16^k-1\right) \vdots 3 \Rightarrow a \vdots 3 \Rightarrow a b \vdots 3 .$

46. Ta có $2 S_n=n(n+1)$.

Mặt khác, sử dụng tính chất $a^n+b^n \vdots(a+b), \forall a, b \in \mathbb{N}^*$ và $n$ lẻ ta có

$2 T_n=\left(1^5+n^5\right)+\left(2^5+(n-1)^5\right)+\ldots+\left(n^5+1\right) \vdots(n+1)\quad\quad(1) .$

$2 T_n=\left(1^5+(n-1)^5\right)+\left(2^5+(n-2)^5\right)+\ldots+\left((n-1)^5+1\right)+2 n^5 \vdots n\quad\quad(2) .$

Do $(n, n+1)=1$, từ $(1)$ và $(2)$ ta suy ra

$2 T_n \vdots n(n+1)=2 S_n \Rightarrow T_n \vdots S_n .$

Tổng quát, ta có thể chứng minh được:

$1^k+2^k+\ldots+n^k$ chia hết cho $1+2+\ldots+n, \forall n, k \in \mathbb{N}, n \geq 1$ và $k$ lẻ.

47. Dễ thấy $n=1$ thỏa mãn và $n=4$ không thỏa mãn. Xét $n>1$ và $n \neq 4$ :

Từ giả thiết suy ra $n$ là hợp số, như vậy $n$ có thể viết được dưới dạng $n=p . q$, trong đó $p, q$ là các số nguyên dương thỏa mãn: $2 \leq p, q \leq\left[\frac{n}{2}\right]$.

  • Nếu $p \neq q$ thì trong tích $(n-1) !=1.2 \ldots n$ chứa cả hai số $p$ và $q$ nên $(n-1)$ ! chia hết cho $n$.

  • Nếu $p=q$ thì $p, q>2$ và trong tích $(n-1)$ ! chứa cả $p$ và $2 p$ nên $(n-1)$ ! chia hết cho $n$.

48. Xem lời giải bài 47.

49. Giả sử $d=\left(\frac{a^m-1}{a-1}, a-1\right)$ và $d^{\prime}=(m, a-1)$. Ta có

$\frac{a^m-1}{a-1}=a^{m-1}+a^{m-2}+\ldots+a+1$

$=\left(a^{m-1}-1\right)+\left(a^{m-2}-1\right)+\ldots+(a-1)+m .$

Vì $a^i-1$ chia hết cho $a-1$ và do đó chia hết cho $d$ với mọi $i \geq 1$ nên $d \backslash m$. Suy ra $d \backslash d$.

Ngược lại, vì $d^{\prime} \backslash a-1$ nên $d^{\prime} \backslash a^i-1$ với mọi $i \geq 1$. Hơn nữa, $d^{\prime} \backslash m$ nên $d^{\prime} \backslash$ $\frac{a^m-1}{a-1}$ và do đó $d^{\prime} \backslash d$.

Vậy $d=d$.

50. $\Rightarrow)$ Giả sử $a^n-1 \backslash a^m-1$. Ta chứng minh $n \backslash m$.

Đặt $m=q n+r, 0 \leq r<n$. Ta có

$a^m-1=a^{n q+r}-1=a^r\left(a^{n q}-1\right)+a^r-1$

$\text { Vì } a^n-1 \backslash a^m-1 \text { và } a^n-1 \backslash a^{n q}-1 \text { nên } a^n-1 \backslash a^r-1 .$

$\text { Mặt khác } 0 \leq a^r-1<a^n-1 \text { nên } a^r-1=0 \text { hay } r=0 .$

$\Leftarrow)$ Dễ dàng chứng minh được.

51. Giả sử $d=(m, n)$ và $d=\left(a^m-1, a^n-1\right)$. Đặt $m=d . m_1, n=d . n_1$ ta có $a^m-1=\left(a^d\right)^{m_1}-1$ chia hết cho $a^d-1$

và $\quad a^n-1=\left(a^d\right)^{n_1}-1$ chia hết cho $a^d-1$

nên d’ chia hết cho $a^d-1\quad\quad(1)$.

Mặt khác, vì $d=(m, n)$ nên tồn tại hai số nguyên dương $x, y$ sao cho $m x-n y=d$. Vì $d^n \backslash a^m-1$ và $d^{\prime} \backslash a^n-1$ nên $d^{\prime} \backslash a^{m x}-1$ và $d^n \backslash a^{n y}-1$.

Suy ra $d^n \backslash a^{m x}-a^{n y}=a^{n y}\left(a^d-1\right)$. Vì $\left(d^n, a^{n y}\right)=1$ nên $d^n \backslash a^d-1\quad\quad(2)$.

Từ (1) và (2) ta có đpcm.

52. Giả sử $m$ không chia hết cho $n$, tức là $m=q \cdot n+r$ với $0<r<n$. Ta có

$a^m+b^m=a^{m-n}\left(a^n+b^n\right)-b^n\left(a^{m-n}-b^{m-n}\right) .$

Suy ra $a^n+b^n \backslash a^{m-n}-b^{m-n}$.

Nếu $q-1>0$, lại làm tương tự như trên tai có

$a^{m-n}-b^{m-n}=a^{m-2 n}\left(a^n+b^n\right)-b^n\left(a^{m-2 n}+b^{m-2 n}\right) .$

Suy ra $a^n-b^n \backslash a^{m-2 n}+b^{m-2 n}$.

Cứ lặp lại cách làm trên ta suy ra $a^n+b^n \backslash a^{m-n \cdot k}+(-1)^k b^{m-n k}, \forall k \leq q$.

Đặc biệt với $k=q$ ta có $a^n+b^n \backslash a^r+(-1)^q b^r$. Điều này không xảy ra vì

$0<\left|a^r+(-1)^q b^r\right|<a^r+b^r<a^n+b^n \text {. }$

53. Ta có $k-b \backslash k^n-a=\left(k^n-b^n\right)+\left(b^n-a\right)$ và $k-b \backslash k^n-b^n$ nên $k-b \backslash b^n-a$. Vì điều này đúng với mọi $k$ nên chọn $k$ sao cho $k-b>\left|b^n-a\right|$. Vì $b^n-a$ chia hết cho $k-b$ nên $b^n-a=0$ hay $a=b^n$.

54. Cần tìm $n$ sao cho $4 n^2+1$ chia hết cho 65 . Đặt $n=65 k+r$, ta chọn $r$ sao cho $4 r^2+1=65$ hay $r=\pm 4$.

Khi đó, mọi số $n$ có dạng $65 k \pm 4$ đều thỏa mãn.

55. Ta có

$\text { Ta có } \frac{p}{q}=\left(1+\frac{1}{2}+\ldots+\frac{1}{1319}\right)-2\left(\frac{1}{2}+\frac{1}{4}+\ldots+\frac{1}{1318}\right)$

$=\left(1+\frac{1}{2}+\ldots+\frac{1}{1319}\right)-\left(1+\frac{1}{2}+\frac{1}{3}+\ldots+\frac{1}{659}\right)$

$=\frac{1}{660}+\ldots+\frac{1}{1319}$

$\Rightarrow 2 \cdot \frac{p}{q}=\left(\frac{1}{660}+\frac{1}{1319}\right)+\left(\frac{1}{661}+\frac{1}{1318}\right)+\ldots+\left(\frac{1}{1319}+\frac{1}{660}\right)=\frac{1979 . A}{B} \text {. }$

Từ đó suy ra $p$ chia hết cho 1979 .

56. Đặt $x_1=a_1 \cdot a_2, x_2=a_2 \cdot a_3, \ldots, x_n=a_n \cdot a_1$.

Vì $a_1, a_2, \ldots, a_n \in{-1,1}$ nên $x_1, x_2, \ldots, x_n \in{-1,1}$.

Ta có $x_1+x_2+\ldots+x_n=0$ suy ra trong các số $x_1, x_2, \ldots, x_n$ số các số 1 bằng số các số $-1$.

Giả sử số các số 1 là $m\left(\mathrm{~m} \in \mathbb{N}^*\right)$ thì $n=2 m$ và

$x_1 x_2 \ldots x_n=(-1)^m \text {. }\quad\quad(1)$

Mặt khác, $x_1 x_2 \ldots x_n=\left(a_1 a_2 \ldots a_n\right)^2=1\quad\quad(2)$.

Từ (1) và (2) suy ra $m$ chã̃n và điều đó có nghĩa là $n$ chia hết cho 4 .

57. Giả sử $p$ số nguyên liên tiếp đó là: $a+1, a+2, \ldots, a+p(\mathrm{a} \in \mathbb{Z})$.

Đặt $A=(a+1)^2+(a+2)^2+\ldots+(a+p)^2$. Ta có

$\mathrm{A}=p \cdot a^2+2(1+2+\ldots+p) \cdot a+\left(1^2+2^2+\ldots+p^2\right)$

Mặt khác: $1+2+\ldots+p=\frac{p(p+1)}{2}, 1^2+2^2+\ldots+p^2=\frac{p(p+1)(2 p+1)}{6}$.

Suy ra $6 A=p\left[6 a^2+6(p+1) a+(p+1)(2 p+1)\right]$ chia hết cho $p$.

Do $p$ là số nguyên tố và $p>3$ nên $(p, 6)=1$. Vậy $A$ chia hết cho $p$.

58. Vì $a^{2001}-a^{2000}=a^{2000}(a-1) \geq 2^{2000}=1024^{200}>10^{600}$ nên giữa $a^{2000}$ và $a^{2001}$ có ít nhất $10^{600}$ số nguyên dương liên tiếp. Trong số đó, tồn tại một số chia hết cho $10^{600}$, đó chính là số $A$ cần tìm.

59. Đáp số: 

Tồn tại. Có thể chọn 2002 số chia hết cho 2003 và 2002 số chia cho 2003 dư 1 .

60. Ta có $(a+b)^7-a^7-b^7=7 a b(a+b)\left(a^2+a b+b^2\right)^2$.

Chọn $b=1$ và $a^2+a+1=7^3$ (bạn đọc tự tính $a$ ) ta có đpcm.

61. Không mất tính tổng quát, giả sử $c=a-b>0$.

Ta có $b=q c+r$, với $q \geq 0,0 \leq r<c, q$ và $r$ không đồng thời bằng 0 .

Các số $n$ có dạng $n=c+1-r+k c, k \in \mathbb{Z}$ đều thoả mãn.